You are on page 1of 66

Dr.

ASHIQ TUTORAL

ONLINE EXAM
PRE KROK 2 MEDICINE

TIMING; 2 HRS 40 MIN

“EXAMS WILL TEST NOT ONLY YOUR KNOWLEDGE BUT ALSO YOUR STATE OF MIND.
SO CALM DOWN AND BE RELAXED. BECAUSE YOU ARE TRULY THE BEST”

NOTE: This is for your self-evaluation purpose, Don’t cheat yourself


1. A 62-year-old woman was brought into the B. Chronic non-ulcerative colitis
admission room with complaints of severe C . Irritable bowel syndrome
burning retrosternal pain and asphyxia. She has D. Crohn disease (regional enteritis)
a 10-year-long history of essential hypertension. E. Whipple disease
Objectively her condition is moderately severe.
4. A 72-year-old man complains of lower
She presents with skin pallor, cyanotic lips, and
extremity edema, sensation of heaviness in the
vesicular respiration over her lungs. The II heart
right subcostal area, dyspnea at rest. For over
sound is accentuated over the aorta. Blood
25 years he has been suffering from COPD.
pressure - 210/120 mm Hg, heart rate (pulse) -
Objectively: orthopnea, jugular venous
76/min. ECG shows elevation of ST segment in
distention, diffuse cyanosis, acrocyanosis.
the leads I, AVL, and V5-V6. What is the most
Barrel chest is observed, on percussion there is
likely diagnosis?
a vesiculotympanitic (bandbox) resonance,
A. Hypertensive crisis complicated with acute
sharply weakened vesicular respiration on both
myocardial infarction
sides, moist crepitant crackles in the lower
B. Uncomplicated hypertensive crisis
segments of the lungs. Heart sounds are
C. Hypertensive crisis complicated with instable
weakened, the II heart sound is accentuated
angina pectoris
over the pulmonary artery. The liver is +3 cm.
D. Hypertensive crisis complicated with acute
What complicated the clinical course of COPD in
left ventricular failure
this patient?
E. Pulmonary embolism
A. Pulmonary embolism
2. A 35-year-old patient developed an epileptic B Chronic pulmonary heart
attack with tonoclonic spasms that lasted for 3 C. Acute left ventricular failure
minutes. After the attack the patient fell asleep D. Diffuse pneumosclerosis
but in 5 minutes the second attack occurred. E. Community-acquired pneumonia
The first step of emergency aid would be to:
A Prescribe antiepileptic drugs 5. A 72-year-old man with pneumonia
B. Take blood from the vein for analysis complains of marked dyspnea, chest pain,
C. Introduce diazepam intravenously severe cough with expectoration, t o is 39.5-
D. Ensure patency of airways 40oC, no urination for a whole day. Objectively
E. Administer chloral hydrate via an enema the patient is conscious. Respiratory rate is
36/min. Over the right lower pulmonary lobe
3. A 27-year-old woman, a teacher in the percussion sound is dull; on auscultation there
elementary school, complains of frequent is bronchial respiration and numerous moist
stools, up to 3 times per day, with lumpy feces crackles. Blood pressure is 80/60 mm Hg. Heart
and large amount of mucus, abdominal pain rate is 120/min. Heart sounds are muffled,
that gradually abates after a defecation, there is tachycardia. What tactics should the
irritability. Her skin is pale and icteric. Pulse is family doctor choose in the management of this
74/min., rhythmic, can be characterized as patient?
satisfactory. Blood pressure is 115/70 mm Hg. A. Hospitalization into the intensive care unit
The abdomen is soft, moderately tender along B. Outpatient treatment
the colon on palpation. Fiberoptic colonoscopy C. Treatment in the day patient facility
detects no changes. What disease can be D. Hospitalization into the pulmonology unit
suspected? E. Hospitalization into the neurology unit
A. Chronic enteritis
6. 2 hours after eating unknown mushrooms, a A. Coarctation of the aorta
28-year-old man sensed a decrease in his B. Aortic stenosis
mobility and deterioration of his ability to focus. C. Pulmonary artery stenosis
This condition was then followed by a state of D .Mitral stenosis .
agitation and agression. On examiantion he is E. Patent ductus arteriosus
disoriented and his speech is illegible. 4 hours
9. A 23-year-old man complains of severe pain
later he developed fetor hepaticus and lost his
in his left knee joint. Objectively the left knee
consciousness. What syndrome can be
joint is enlarged, with hyperemic skin, painful
observed in this patient?
on palpation. Complete blood count:
A. . Cytolytic syndrome
erythrocytes - 3.8 · 1012/L, Hb- 122 g/L,
B. Hepatolienal syndrome
leukocytes - 7.4 · 109/L, platelets - 183 · 109/L.
C. Portal hypertension
Erythrocyte sedimentation rate - 10 mm/hour.
D. Cholestatic syndrome
Bleeding time (Duke method) - 4 min., Lee-
E Acute hepatic failure
White coagulation time - 24 min. Partial
7. A 36-year-old man complains of marked thromboplastin time (activated) - 89 seconds.
dyspnea and cardiac pain. He ascribes his Rheumatoid factor - negative. What is the most
disease to the case of influenza that he had 2 likely diagnosis?
weeks ago. Objectively he leans forward when A. . Werlhof disease (immune
sitting. The face is swollen, cyanotic, cervical thrombocytopenia)
veins are distended. Heart borders are B. Hemophilia, hemarthrosis
extended on the both sides, heart sounds are C. Rheumatoid arthritis
muffled, heart rate = Ps = 118/min., BP is 90/60 D. Thrombocytopathy
mm Hg. Blood test: ESR is 46 mm/hour. ECG E. Hemorrhagic vasculitis (Henoch-Schonlein
shows low voltage. Xray shows trapezoidal purpura), articular form
cardiac silhouette and signs of pulmonary
10. A 24-year-old woman, a kindergarten
congestion. Choose the treatment tactics:
teacher, has been sick for 2 days already.
A. Pericardial puncture (pericardiocenthesis)
Disease onset was acute. She presents with
B. Diuretics
elevated body temperature up to 38.0oC, pain
C. Antibiotics
attacks in her lower left abdomen, liquid stool
D. Pericardectomy
in small amounts with blood and mucus
E. Glucocorticosteroids
admixtures 10 times a day. Pulse - 98/min.,
8. A 39-year-old man suffers from chronic blood pressure - 110/70 mm Hg. Her tongue is
rheumatic heart disease. He complains of moist and coated with white deposits. The
dyspnea during physical exertion, cough with abdomen is soft, the sigmoid colon is painful
expectoration, and palpitations. Ausculation and spastic. Make the provisional diagnosis:
detects intensified I heart sound and diastolic A. . Salmonellosis
murmur; the sound of opening mitral valve can B. Escherichiosis
be auscultated at the cardiac apex. The II heart C. Shigellosis
sound is accentuated over the pulmonary D. Yersiniosis
artery. The patient is cyanotic. X-ray shows E. Rotavirus infection
dilated pulmonary root and enlargement of the
11. A pregnant woman is 28 years old.
right ventricle and left atrium. What is the most
Anamnesis: accelerated labor complicated by
likely diagnosis?
the II degree cervical rupture. The following two
pregnancies resulted in spontaneous abortions curettage of the uterine cavity yields a
at the terms of 12 and 14 weeks. On mirror significant amount of medullary substance in
examination: the uterine cervix is scarred from the scrape. What is the most likely diagnosis?
previous ruptures at 9 and 3 hours, the cervical A. Chorioepithelioma
canal is gaping. On vaginal examination: the B. Adenomyosis
cervix is 2 cm long, the external orifice is open 1 C. Uterine corpus cancer
cm wide, the internal orifice is half-open; the D. Uterine cervix cancer
uterus is enlarged to the 12th week of E. Hormone-producing ovarian tumor
pregnancy, soft, mobile, painless, the
14. A 48-year-old woman complains of
appendages are without changes. What
disturbed menstrual cycle: her periods last for
diagnosis can be made?
7-9 days and are excessively profuse
A. Isthmico-cervical insufficiency, habitual
throughout the last half-year. She notes
noncarrying of pregnancy
occasional hot flashes in her head, insomnia,
B. Threatened spontaneous abortion
irritability, and headaches. Her skin is of normal
C. Incipient abortion, habitual noncarrying of
color. Blood pressure - 150/90 mm Hg, pulse -
pregnancy
90/min., rhythmic. The abdomen is soft and
D. Cervical hysteromyoma, habitual noncarrying
painless. Bimanual examination shows no
of pregnancy
uterine enlargement, the appendages cannot
E. Cervical pregnancy, 12 weeks
be detected. The vaginal fornices are free. What
12. On the day 4 after the cesarean section a is the most likely diagnosis?
woman developed fever with body temperature A. Climacteric syndrome
up to 39oC and abdominal pain. Pulse - B. Premenstrual syndrome
104/min. She vomited twice. The patient is C. Adrenogenital syndrome
sluggish, her tongue is dry and has gray coating. D. Stein-Leventhal syndrome (polycystic ovary
The abdomen is distended. Signs of peritoneal syndrome)
irritation are positive in all segments. Peristalsis E. Uterine myoma
cannot be auscultated. No passage of gas
15. A 30-year-old multigravida has been in
occurs. Uterine fundus is located at the level of
labour for 18 hours. 2 hours ago the pushing
the navel. The uterus is painful on palpation.
stage began. Fetal heart rate is clear, rhythmic,
The discharge is moderate and contains blood
136/min. Vaginal examination reveals complete
and pus. What is the most likely diagnosis?
cervical dilatation, the fetal head in the pelvic
A. . Parametritis
outlet plane. Sagittal suture is in line with
B. Metroendometritis
obstetric conjugate, the occipital fontanel is
C. Progressive thrombophlebitis
near the pubis. The patient has been diagnosed
D. Pelvic peritonitis
with primary uterine inertia. What is the further
E. Diffuse peritonitis
tactics of labor management?
13. A 58-year-old woman came to the A. Vacuum extraction of the fetus
gynecological clinic. She complains of bloody B. Labour stimulation
discharge from her genital tracts. Menopause is C. Cesarean section
8 years. Gynecological examination: the uterus D. Skin-head Ivanov’s forceps
is slightly enlarged, dense to touch, with limited E. Outlet forceps
mobility; the uterine appendages cannot be
16. A woman is 40 weeks pregnant. The fetus is
detected; parametrium is free. Fractional
in the longitudinal lie and cephalic presentation.
Pelvic size: 26-29-31- 20. Expected weight of the gynecologist choose?
fetus is 4800 gram. The labor contractions has A. Abortion
been lasting for 12 hours, within the last 2 B. Prescription of antibacterial therapy
hours they were extremely painful, the C. Prescription of antiviral therapy
parturient woman is anxious. The waters broke D. Treatment of incipient abortion
4 hours ago. On external examination the E. Prescription of hemostatic therapy
contraction ring is located 2 finger widths above
19. A 9-month-old child presents with fever,
the navel, Henkel-Vasten sign is positive. Fetal
cough, dyspnea. The symptoms appeared 5
heart rate is 160/min., muffled. On internal
days ago after a contact with a person with
examination the uterine cervix is fully open, the
URTI. Objectively: the child is in grave condition.
head is engaged and pressed to the entrance
Temperature is 38oC, cyanosis of nasolabial
into the lesser pelvis. What is the most likely
triangle is present. RR- 54/min, nasal flaring
diagnosis?
during breathing is observed. There was
A. Abruption of the normally positioned
percussion dullness on the right below the
placenta
scapula angle and tympanic sound over the
B. Complete uterine rupture
other areas of lungs. Auscultation revealed
C. Hyperactive uterine contractions
bilateral fine moist crackles predominating on
D .Threatened uterine rupture
the right. What is the most likely diagnosis?
E. Anatomically contracted pelvis
A. Acute bronchiolitis
17. A 23-year-old woman came to the B. URTI
gynecological clinic. She complains of pain, C. Acute laryngotracheitis
itching, and burning in her vulva, general D. Acute bronchitis
weakness, indisposition, elevated body E. Pneumonia
temperature up to 37.2oC, and headache. On
20. The mother of a 3-month-old child came to
examination in the vulva there are multiple
a family doctor with complaints of her child
vesicles up to 2-3 mm in diameter with clear
being physically underdeveloped and suffering
contents against the background of hyperemia
from cough attacks and dyspnea. Anamnesis:
and mucosal edema. Make the provisional
the child is the result of the second full-term
diagnosis:
pregnancy with the risk of miscarriage (the first
A. Papillomavirus infection
child died of pulmonary pathology at the age of
B. Primary syphilis
4 months, according to the mother). Body mass
C. Genital herpes infection
at birth is 2500 g. Cough attacks were observed
D. Vulvar cancer
from the first days of life, twice the child was
E. Cytomegalovirus infection
treated for bronchitis. Considering the severity
18. A woman with the pregnancy term of 8 of the child’s condition the doctor made the
weeks complains of elevated temperature up to referral for hospitalization. What diagnosis was
37.6oC, skin rash that can be characterized as most likely stated in the referral?
macular exanthema, enlargement of posterior A. Recurrent obstructive bronchitis
cervical and occipital lymph nodes, small B. Acute obstructive bronchitis
amount of bloody discharge from the genital C .Mucoviscidosis (Cystic fibrosis)
tracts. She was examined by the infectious D. Pertussis
diseases specialist and diagnosed with rubella. E. Acute obstructive pneumonia
What tactics should the obstetrician-
21. A 46-year-old man notes swollen legs, C. Varicocele
weakness, sensation of fullness and heaviness D. Dystopic kidney
in the right subcostal area; it is the first E. Necrotic papillitis
occurrence of these signs in the patient. The
24. A man complains of constant dull pain in the
patient has 20-year-long history of rheumatoid
perineum and suprapubic area, weak flow of
arthritis. The liver and spleen are enlarged and
urine, frequent difficult painful urination,
dense. Blood creatinine - 0,23 mmol/l,
nocturia. The patient has been suffering from
proteinemia - 68 g/l, cholesterol - 4,2 mmol/l,
this condition for several months, during which
urine specific gravity - 1012, proteinuria - 3,3
urination was becoming increasingly difficult,
g/l, isolated wax-like cylinders, leached
and pain in the perineum has developed. On
erythrocytes in the vision field, leukocytes - 5-6
rectal examination: the prostate is enlarged
in the vision field. What is the most likely
(mainly its right lobe), dense, asymmetrical,
complication?
central fissure is smoothed out, the right lobe is
A. Chronic glomerulonephritis
of stony density, painless, tuberous. What
B. Renal amyloidosis
disease is it?
C. Acute glomerulonephritis
A. Prostate tuberculosis
D. Heart failure
B. Prostate sclerosis
E. Chronic pyelonephritis
C. Urolithiasis, prostatolith of the right lobe
22. A 23-year-old man had taken 1 g of aspirin D. Prostate cancer
to treat acute respiratory infection. After that E. Chronic congestion prostatitis
he developed an asthmatic fit with labored
25. A boy was born at 32 weeks of gestation. 2
expiration that was arrested by introduction of
hours after the birth he developed respiratory
aminophylline. The patient has no medical
distress (RD). The RD severity assessed by
history of allergies. The patient has undergone
Silverman score was 5. The respiratory
two surgeries for nasal polyposis in the past.
disorders progressed, respiratory failure could
What is the most likely diagnosis?
not be eliminated by Martin-Bouyer CPAP
A. . Symptomatic bronchospasm
(continuous positive airway pressure). X-ray of
B. Atopic bronchial asthma
lungs shows reticular and nodular pattern, air
C. Infectious allergic bronchial asthma
bronchogram. What is the most likely cause of
D. Exercise-induced asthma
respiratory distress syndrome?
E. Aspirin-induced asthma
A. Bronchopulmonary dysplasia
23. A 50-year-old patient was delivered to a B. Segmental atelectasis
hospital with complaints of blood traces in C. Hyaline membrane disease
urine. Urination is painless and undisturbed. D. Congenital pulmonary emphysema
Macrohematuria had been observed for 3 days. E. Edematous hemorrhagic syndrome
Objectively: kidneys cannot be palpated,
26. A 9-year-old girl complains of fever up to
suprapubic area is without alterations, external
37,5oC, headache, inertness, weakness, loss of
genitalia are non-pathologic. On rectal
appetite, stomachache, and frequent painful
investigation: prostate is not enlarged, painless,
urination. Provisional diagnosis of acute
has normal structure. Cystoscopy revealed no
pyelonephritis is made. Clinical urine analysis:
alterations. What is the most likely diagnosis?
specific gravity - 1018, no protein, leukocytes -
A. Renal carcinoma
10-15 in the vision field. What investigation
B. Bladder tuberculosis
method can verify the diagnosis of urinary
system infection? 30. A 32-year-old woman complains of body
A. Bacteriological inoculation of urine weight loss despite her increased appetite,
B. Rehberg test (creatinine clearance test) nervousness, and tremor of the extremities.
C. Zymnytsky test (measurement of daily Objectively: the skin is moist; the thyroid gland
diuresis) is diffusely enlarged, painless, soft, and mobile.
D. Complete blood count Blood test: increased level of T3, T4, and
E. Clinical urine analyses, dynamic testing thyroid-stimulating hormone (THS). What is the
most likely diagnosis?
27. During assessment of work conditions at the
A. Diffuse toxic goiter
mercury thermometer manufacture, content of
B. Thyroid carcinoma
mercury vapors in the air of working area is
C. Autoimmune (Hashimoto’s) thyroiditis
revealed to exceed maximum concentration
D. Thyroid adenoma
limit. Specify the main way of mercury
E. Diffuse nontoxic goiter
penetration into the body:
A. Intact skin 31. A 57-year-old patient complains of
B. Respiratory organs sensation of dryness and pain during
C. Damaged skin swallowing, frequent unbearable cough, the
D. Gastrointestinal tract voice is hoarse. Disease onset was abrupt. On
E. Mucous tunics laryngoscopy: laryngeal mucosa is hyperemic,
vocal folds are swollen, laryngeal lumen
28. During health assessment of car drivers and
contains viscous secretion. What diagnosis is it?
police officers on point duty, the physicians
A. Acute stenosing laryngotracheitis
detected carboxyhemoglobin in the blood of
B. Acute laryngitis
the patients, weakened reflex responses,
C. Bronchial asthma
disturbed activity of a number of enzymes.
D. Flegmonous laryngitis
Revealed professional health disorders are most
E. Laryngeal diphtheria
likely to be associated with the effect of:
A. Nitric oxide 32. A 24-year-old pregnant woman on her 37th
B. Sulfurous anhydride week of pregnancy has been delivered to a
C. Mental stress maternity obstetric service with complaints of
D. Aromatic hydrocarbons weak fetal movements. Fetal heartbeats are
E. Carbon monoxide 95/min. On vaginal examination the uterine
cervix is tilted backwards, 2 cm long, external
29. On the 3rd day of life a newborn, who had
orifice allows inserting a fingertip. Biophysical
suffered birth asphyxia, developed hemorrhage
profile of the fetus equals 4 points. What tactics
from the umbilical wound. Laboratory analysis
of pregnancy management should be chosen?
reveals hypocoagulation, thrombocytopenia,
A. . Urgent preparation of the uterine cervix for
and hypothrombinemia. What is the cause of
delivery
such clinical developments?
B. Treatment of placental dysfunction and
A. Congenital angiopathy
repeated analysis of the fetal biophysical profile
B. Hemorrhagic disease of newborn
on the next day
C. Disseminated intravascular coagulation
C. Doppler measurement of blood velocity in
D. Thrombocytopenic purpura
the umbilical artery
E. Umbilical vessel trauma
D Urgent delivery via a cesarean section
E. Treatment of fetal distress, if ineffective, then C. Total protein content in the pleural fluid
elective cesarean section on the next day above 30 g/l
D. Specific gravity exceeding 1015
33. During regular preventive gynecological
E. Total protein content in the pleural fluid
examination a 30-year-old woman was detected
below 25 g/l
to have dark blue punctulated ”perforations” on
the vaginal portion of the uterine cervix. The 36. A 28-year-old woman complains of girdle
doctor suspects endometriosis of the vaginal pain in her epigastric and left subcostal areas
portion of the uterine cervix. What investigation with irradiation to the back, nausea, and
method would be most informative for vomiting without relief. On examination a
diagnosis confirmation? surgeon observes stomach distension and
A. Colposcopy, target biopsy of the cervix meteorism. There are positive Mondor’s, Mayo-
B. US of small pelvis Robson’s, and Cullen’s symptoms. What is the
C. Hysteroscopy most likely diagnosis?
D. Curettage of the uterine cavity A Aortic dissecting aneurysm
E. Hormone testing B. Acute cholecystitis
C. Acute intestinal obstruction
34. A 26-year-old woman came to a
D. Acute pancreatitis
gynecologist for a regular check-up. She has no
E. Splenic infarction
complaints. Per vaginum: the uterus lies in
anteflexion, not enlarged, dense, mobile, 37. A 36-year-old woman complains of pain in
painless. On the left from the uterus in the area her lumbar area, which irradiates to her lower
of uterine appendages there is a mobile right limb and increases during movements, and
painless outgrowth that can be moved sensation of numbness in her limb. Objectively:
independently from the uterus. On the right the palpation of the shin and thigh muscles is
appendages cannot be detected. What painful, positive stretch symptom on the right.
additional investigation would be informative MRI scan: herniation of intervertebral disk L5-S1
for diagnosis clarification? 4 mm in size. What is the most likely diagnosis?
A. Metrosalpingography A. Vertebrogenic lumbago
B. US of lesser pelvis B. Vertebrogenic radicular syndrome of L5-S1
C. Examination for urogenital infection on the right
D. Colposcopy C. Endarteritis of lower extremities
E. Colonoscopy D. Spinal stroke
E. Acute myelitis
35. A 57-year-old patient complains of dyspnea
at rest. The patient presents with orthopnea, 38. During routine medical examination a 35-
acrocyanosis, bulging cervical veins. On year-old woman presents with enlarged cervical
percussion: dull sound over the lower lung and mediastinal lymph nodes. Her overall
segments; on auscultation: no respiratory health is satisfactory. ESR is 30 mm/hour.
murmurs. Heart rate is 92/min. Right-sided Cervical node biopsy was performed. In the
cardiac dilatation is observed. The liver is specimen there are granulomas composed of
enlarged by 7 cm. Shins are swollen. Pleural epithelial and giant cells, no caseous necrosis
effusion is suspected. What indicator would detected. What is the most likely diagnosis?
confirm the presence of transudate in this case? A. Infectious mononucleosis
A. Positive Rivalta’s test B. Lymphogranulomatosis
B. Presence of atypical cells C. Sarcoidosis
D. Nonspecific lymphadenitis 42. A 17-year-old girl has made an appointment
E. Lymph node tuberculosis with the doctor. She plans to begin her sex life.
No signs of gynecological pathology were
39. A 39-year-old woman complains of violent
detected. In the family history there was a case
pain in her left lumbar area with irradiation to
of cervical cancer that occurred to the patient’s
the right iliac area. Several years ago she was
grandmother. The patient was consulted about
diagnosed with cholelithiasis and urolithiasis.
the maintenance of her reproductive health.
The patient’s condition is moderately severe,
What recommendation will be the most helpful
the skin is dry. Ortner’s symptom is negative;
for prevention of invasive cervical cancer?
costovertebral angle tenderness is observed on
A. Vaccination against human papillomavirus
the right. The most reasonable treatment
(HPV)
tactics would be
B. Vitamins, calcium, omega-3 C.
A Urgent hemodialysis
Immunomodulators
B. Spasmolytics and analgesics
D. Antiviral and antibacterial drugs
C. Peritoneal dialysis
E. Timely treatment of sexually transmitted
D. Laparoscopic cholecystectomy
diseases
E. Antibiotics
43. A 6-month-old infant is not vaccinated. The
40. An 18-year-old patient complains of skin
physician recommends a DPT (diphtheria,
rash. The patient has been suffering from this
pertussis, tetanus) vaccination but the mother
condition for 5 years. The first instance of this
is absolutely against this procedure. Choose the
disease occurred after a car accident.
most substantial argument in favor of
Objectively: the patient presents with papular
vaccination:
rash covered in silvery scales, ”thimble”
A. Risk of lethal consequences
symptom (small pits on the nails), affected
B. Epidemic risk for the others
joints. What is the most likely diagnosis?
C. Personal professional experience
A. Onychomycosis
D. High quality of vaccines
B.. Panaritium
C. Psoriasis 44. In autumn a 45-year-old man was
D. Lupus erythematosus recommended an elective surgery for coronary
E. Rheumatism artery bypass grafting due to multivessel
coronary artery disease. The patient has never
41. A 20-year-old student was brought to the
received anti-influenza vaccination. Why would
first-aid center. He has a closed fracture of the
the family doctor offer a scheduled yearly
left forearm and a contused lacerated wound
vaccination against influenza to this patient?
on his left shin. After the patient received initial
A. Prevention of seasonal influenza
wound management, he presented the
B. Secondary prevention of exacerbations of
documents confirming that he has received all
chronic ischemic heart disease
the necessary preventive vaccination as
C. Immunoprophylaxis of postoperative
scheduled. What should the doctor do to
pulmonary complications
prevent tetanus in this patient?
D. Primary prevention of influenza during
A. Antibiotic therapy
postoperative care
B. Administration of tetanus immunoglobulin
E. Decrease the risk of mortality due to
C. Administration of anti-tetanus serum
pneumonia and heart failure
D. Dynamic case monitoring
E. Administration of tetanus toxoid
45. A 28-year-old woman has made an 48. Among first-year schoolchildren there was a
appointment with the family doctor to receive case of measles registered. A 7-year-old boy
vaccination against influenza. However, having from the same group was not vaccinated
collected the patient’s medical history, the against measles due to refusal of his parents.
doctor claimed this procedure to be absolutely His clinical history has no cases of measles in
contraindicated for this woman. What the past and is not contraindicatory to
anamnestic data is the absolute immunobiological agents. Choose the most
contraindication to vaccination? rational tactics of measles prevention in this
A Blood hemoglobin - 109 g/L schoolboy:
B. Pregnancy at 30 weeks A Immunomodulators
C. Egg white intolerance B. Isolation for 20 days
D. Body temperature - 37.2oC C. Antiviral agents
E. Psoriasis in the remission phase D. Antibiotics
E. Measles-Mumps-Rubella vaccine
46. A 26-year-old man complains of chills,
rhinitis, dry cough, and fever up to 38oC. 49. A 40-year-old man developed fever up to
Examination shows him to be in a moderately 37.5oC and macular rash 10 days after the first
severe condition; there are small pale pink non- dose of MMR (Measles-Mumps-Rubella)
merging spots on the skin of his back, abdomen, vaccine was administered. The vaccination was
and extremities. Palpation reveals enlarged considered necessary as there was a measles
occipital and axillary lymph nodes. No outbreak in the city and the patient had not
information about vaccination history could be received MMR vaccination in his childhood. Is
obtained. What is the likely etiology of this revaccination with MMR vaccine possible?
disease? A Simultaneously with antihistamines
A Epstein-Barr virus B. Forbidden
B. Rubella virus C. After a course of glucocorticoids treatment
C. Streptococcus D. Possible
D. Mumps virus E. Under supervision in the infectious diseases
E. Neisseria meningitis inpatient ward

47. During administration of planned DPT 50. A 26-year-old man is undergoing a regular
vaccination the child suddenly developed acute check-up. One year ago he had a case of
anxiety, signs of pain response, dyspnea, tonsillar diphtheria complicated with
grunting respirations, cutis marmorata, cold myocarditis. Presently his condition is
sweat. Objectively the child’s consciousness is satisfactory, no signs of cardiovascular failure;
disturbed, heart rate is 150/min., blood ECG shows firstdegree atrioventricular block.
pressure is 60/40 mm Hg, heart sounds are What vaccine was administered to this man
muffled. The child was diagnosed with according to his age?
anaphylactic shock. What drug should be A Acellular DPT vaccine
administered first? B. Adsorbed diphtheria tetanus vaccine
A. Epinephrine (modified)
B. Lasix (Furosemide) C. Tetanus anatoxin
C. Suprastin (Chloropyramine) D. Oral polio vaccine (OPV)
D. Euphylline (Aminophylline) E. BCG vaccine
E. Analgin (Metamizole)
51. During analysis of morbidity in the city, it 55. On laboratory investigation of a pork sample
was determined that age structure of there is 1 dead trichinella detected in 24
population is different in each district. What sections. This meat should be:
statistical method allows to exclude this factor, A. Sent for technical disposal
so that it would not skew the morbidity data? B. Allowed for sale with no restrictions
A. Standardization C. Processed and sold through public catering
B. Wilcoxon signed-rank test network
C. Correlation-regression analysis D. Processed for boiled sausage production
D. Dynamic time series analysis E. Frozen until the temperature of -10oC is
E. Analysis of average values reached in the deep layers, with subsequent
exposure to cold for 15 days
52. Clinical statistical investigation was
performed to determine effectiveness of a new 56. To assess the effectiveness of medical
pharmacological preparation for patients with technologies and determine the power and
ischemic heart disease. What parametric test direction of their effect on the public health
(coefficient) can be used to estimate the indicators, the research was conducted to study
reliability of the results? the immunization rate of children and measles
A. Wilcoxon signed-rank test incidence rate by district. What method of
B. Sign test statistical analysis should be applied in this
C. Matching factor case?
D. Student’s t-distribution A. Calculation of correlation coefficient
E. Kolmogorov-Smirnov test B. Calculation of morbidity index among the
nonvaccinated
53. In a rural health care area there is an
C. Calculation of matching factor
increasing cervical cancer morbidity observed.
D. Calculation of standardized ratio
The decision is made to conduct a medical
E. Calculation of statistical significance of the
examination of the women living in this locality.
difference between two estimates
What type of medical examination is it?
A. Regular 57. Having studied the relationship between the
B. Preliminary distance from villages to the local outpatient
C. Target clinics and frequency of visits to the clinics
D. Complex among the rural population of this area, it was
E. Screening determined that the rank correlation coefficient
in this case equals -0.9. How can this
54. In the process of hiring, a prospective
relationship be characterized?
employee has undergone preventive medical
A. Strong inverse relationship
examination and was declared fit to work in this
B. Strong direct relationship
manufacturing environment. What type of
C. Moderate inverse relationship
preventive medical examination was it?
D. Moderate direct relationship
A. Comprehensive
B. Scheduled 58. In the inpatient gynecological unit within a
C. Periodical year 6500 women underwent treatment. They
D. Specific spent there a total of 102000 bed-days. What
E. Preliminary indicator of the gynecological unit work can be
calculated based on these data?
A. Bed turnover rate
B. Average bed occupancy rate per year D. Aminocapronic acid
C. Number of beds by hospital department E. Vicasol (Menadione)
D. Average length of inpatient stay
62. A 67-year-old man complains of dyspnea on
E. Planned bed occupancy rate per year
exertion, attacks of retrosternal pain, dizziness.
59. A middle school teacher with 4-yearlong He has no history of rheumatism. Objectively:
record of work was issued a medical certificate pale skin, acrocyanosis. There are crackles in
for pregnancy and childbirth leave. What the lower lungs. There is systolic thrill in the II
amount of pay will she receive for the duration intercostal space on the right, coarse systolic
of her leave in this case? murmur conducted to the vessels of neck. BP-
A. 50% of average salary 130/90 mm Hg, heart rate - 90/min., regular
B.100% of average salary rhythm. The liver extends 5 cm from under the
C. 70% of average salary edge of costal arch, shin edemas are present.
D. 60% of average salary Specify the suspected valvular defect:
E. 80% of average salary A. Ventricular septal defect
B. Pulmonary artery stenosis
60. In the air of the feed kitchen at poultry
C. Mitral insufficiency
factory, at the area where formula feed is being
D .Aortic stenosis
mixed, the dust concentration reaches 200
E. Tricuspid regurgitation
mg/m3. Air microflora is represented
63. A 24-year-old female teacher complains of
predominantly by Asperqillus and Mucor fungi.
dizziness and heart pain irradiating to the left
What effect determines pathogenic properties
nipple. Pain is not associated with physical
of the dust?
activity and cannot be relieved by nitroglycerin,
A. . Mutagenic
it abates after taking Valocordin and lasts an
B. Teratogenic
hour or more. The patient has a nearly 2-year
C. Allergenic
history of this disease. Objectively: Ps- 76/min.,
D. Fibrogenic
BP110/70 mm Hg. Heart borders are normal,
E. Toxic
heart sounds are clear. The ECG shows
61. A 16-year-old adolescent has been respiratory arrhythmia. Radiograph of the
hospitalized with complaints of unceasing nasal cervicothoracic spine reveals no pathology.
hemorrhage and unbearable pain in his right Lungs, abdomen are unremarkable. What
elbow joint. Objectively: the large joint is changes in blood formula can be expected?
enlarged and defigurated, the skin over the A. Increased ESR
joint is hyperemic. Arthropathy signs can be B. Leukocytosis
observed in the other joints. Ps- 90/min. Blood C. Thrombocytopenia
test: erythrocytes - 3, 9 · 1012/l, Нb- 130 g/l, D. Leukemic hiatus
color index - 1,0, leukocytes - 5, 6 · 109/l, E. No changes
platelets - 220 · 109/l, ESR- 6 mm/hour. Lee-
64. A 51-year-old female patient complains of
White coagulation time: start- 24min , end27
frequent defecation and liquid blood-streaked
min10 sec. What drug would be most efficient
stools with mucus admixtures, diffuse pain in
in the treatment of this patient?
the inferolateral abdomen, 6 kg weight loss
A. Cryoprecipitate
within the previous month. Objectively: body
B. Calcium chloride
temperature - 37, 4oC, malnutrition, skin is pale
C. Concentrated red cells
and dry. Abdomen is soft, sigmoid is painful and
spasmodic, makes a rumbling sound. Liver is C. Dexamethasone
dense, painful, extends 3 cm below the costal D. Lasolvan (Ambroxol)
margin. What is the most likely diagnosis? E. Salbutamol
A. Helminthic invasion
68. A newborn with gestational age of 31 weeks
B. Bacillary dysentery presents with hypotonia and depressed
C. Sprue consciousness. Hematocrit is 35%,
D. Intestinal enzymopathy cerebrospinal fluid analysis shows increased
E. Non-specific ulcerative colitis content of erythrocytes and protein, and low
glucose. These data correspond with the clinical
65. Chief physician of a polyclinic charged a
presentation of:
district doctor with a task to determine the
A. Meningitis
pathological prevalence of disease N in his
B. Intracranial hemorrhage
district. What document allows to estimate the
C. Sepsis
disease prevalence in the population of a
D. Anemia
medical district?
E. Prenatal infection
A. .Statistic coupons (-)
B. Statistic coupons (+) 69. A 16-year-old girl has primary amenorrhea,
C Prophylactic examinations register no pubic hair growth, normally developed
D. Statistic coupons (+) and (-) mammary glands; her genotype is 46 ХY; uterus
E. Vouchers for medical appointments and vagina are absent. What is your diagnosis?
A. Sheehan syndrome
66. A 32-year-old woman complains of
B. Mayer-Rokitansky-Kuster-Hauser syndrome
dizziness, headache, palpitation, tremor. For the
C. Cushing syndrome
last several months she has been under
D. Testicular feminization syndrome
outpatient observation for increased arterial
E. Cushing disease
pressure. Since recently such attacks have
become more frequent and severe. Objectively: 70. 6 hours ago the waters of a 30-year-old
the skin is covered with clammy sweat, tremor gravida 1, para 0, burst; her preliminary period
of the extremities is present. HR110/min., BP- was pathologic and lasted for over 2 days; the
220/140 mm Hg. Heart sounds are muffled. term of pregnancy is 39 weeks. No labor activity
Blood test results: WBCs- 9, 8 · 109/l, ESR- 22 is observed. Fetal head presents above the
mm/hour. Blood glucose - 9,8 millimole/l. What pelvic inlet. Fetal heartbeats are 142/min., clear
disease is the most likely cause of this crisis? and rhytmic. On vaginal examination the
A. Preeclampsia uterine cervix is not dilated. What further
B. Essential hypertension tactics should the doctor choose?
C. Pheochromocytoma A. Stimulate the labor with oxytocin
D. Primary hyperaldosteronism B. Induce cervical dilation with prostaglandins
E. Diabetic glomerulosclerosis C .Perform cesarean section
D. Wait for the onset of spontaneous labor
67. A 14-year-old boy presents with moderate
E. Prolong the pregnancy, while providing
bronchial asthma in its exacerbation period.
antibacterial treatment
What drug should be prescribed to stop an
acute attack of expiratory dyspnea? 71. Mother of an 8-year-old girl complains that
A. Strophanthine (cardiac glycosides) the child is too short and has excessive body
B. Cromolyn sodium (Cromoglicic acid) weight. Objectively: obesity with fat deposits on
the torso and face (round moon-like face), acne, E . Veloergometry
striae on the thighs and lower abdomen,
75 A 59-year-old male patient with essential
hirsutism. What hormone can cause such
hypertension of stage II is registered with the
symptoms, when in excess?
dispensary department of a polyclinic. The
A. Glucagon
patient regularly takes ACE inhibitors and
B. Thyroxin
calcium antagonists. How often should a
C. Testosterone therapeutist examine this patient (except for
D. Insulin exacerbation periods)?
E. Cortisol
A. Every 6 months
72. An emergency team deliverd a 83year-old B. Every 3 months
patient complaining of inability of her right leg C. Every 4 months
to support the body after falling on her right D. Once a year
side. Objectively: the patient lies on a gurney,
E. Every 9 months
her right leg is rotated outwards, the outside
edge of foot touches the bed. There is positive
straight leg raising sign. What is your provisional
diagnosis? 76. A 47-year-old woman came to the
admission room with complaints of general
A. Femoral neck fracture weakness, dizziness, vomiting with blood clots.
B. Femoral diaphysis fracture Condition onset was 3 hours ago. The patient
C. Hip dislocation has no preceding illnesses. Blood pressure is
D. Hip joint contusion 90/60 mm Hg, pulse is 106/min., of poor
E. Cotyloid cavity fracture volume. The abdomen is soft, with mild
tenderness in the epigastrium. Blood test:
73. A newborn has Apgar score of 9. When
erythrocytes - 2.1·1012/L, Нb- 70 g/L,
should the infant be put to the breast?
hematocrit - 28%. What tactics should the
A. After 2 hours
doctor on duty choose?
B. After 12 hours
A. Refer the patient to the family doctor
C. In the delivery room
B. Consult the surgeon
D. On the 2nd day
C. Give spasmolytics
E. On the 3rd day
D. Perform gastric lavage
74. A hospital admitted an 11-yearold boy E. Make an appointment for colonoscopy
diagnosed with medium-severe asthma,
77. A 45-year-old woman complains of
exacerbation period. In order to arrest the
paroxysmal intolerable facial pain on the left
attacks the boy was administered
with attacks that last for 1-2 minutes. Attacks
broncholytic nebulizer therapy. During the
day the child’s condition stabilized. What is are provoked by chewing. The disease onset
the most appropriate method for further was two month ago after overexposure to cold.
monitoring of respiratory function in this Objectively: pain at the exit points of the
patient? trigeminal nerve on the left. Touching near the
wing of nose on the left induces new pain
A. Peak flowmetry attack with tonic spasm of the facial muscles.
B. Spirometry What is the most likely diagnosis among those
C. Pneumotachometry listed?
D. Bronchodilatation tests A. Maxillary sinusitis
B.Glossopharyngeal neuralgia A. . Rendu-Osler-Weber disease (Hereditary
C. Temporomandibular joint arthritis hemorrhagic telangiectasia)
D. Facial migraine B. Hemophilia
E. Trigeminal neuralgia C. Idiopathic thrombocytopenic purpura
D. Disseminated intravascular coagulation
78. A 10-year-old boy with symptoms of
E. Acute vascular purpura
arthritis and myocarditis was delivered into a
hospital. Based on clinical examination the 81. A 7-year-old boy has been an inpatient for
preliminary diagnosis of juvenile rheumatoid 1.5 months. He had been delivered to the
arthritis was made. What symptom is the most hospital with complaints of edemas all over his
contributive for the diagnostics of this disease? body, low urine output, and headache. Clinical
A. Reduced mobility of the joints in the morning urinalysis: proteins - 7.1 g/L, leukocytes - 1-2 in
B. Regional hyperemia of the joints the vision field, erythrocytes - 3-4 in the vision
C. Affection of the large joints field. During the course of treatment the
D. Enlarged heart edemas gradually dissipated, headache abated,
E. Increased heart rate diuresis normalized. Daily urine proteins - 3 g/L.
Biochemical blood test: total protein - 43.2 g/L,
79. A 25-year-old patient was delivered to an
urea - 5.2 mmol/L, cholesterol - 9.2 mmol/L.
infectious diseases unit on the 3rd day of illness
What glomerulonephritis syndrome is the most
with complaints of headache, pain in lumbar
likely to be present in the patient?
spine and gastrocnemius muscles, high fever,
A. Nephritic
chill. Objectively: condition of moderate
B. Nephrotic
severity. Scleras are icteric. Pharynx is
C. Isolated urinary
hyperemic. Tongue is dry with dry brown
D. Hematuric
coating. Abdomen is distended. Liver is
E. Mixed
enlarged by 2 cm. Spleen is not enlarged.
Palpation of muscles, especially gastrocnemius 82. 2 weeks after labour a parturient woman
muscles, is painful. Urine is dark in colour. Stool developed breast pain being observed for 3 days.
is normal in colour. The most likely diagnosis is: Examination revealed body temperature at the
A. Malaria rate of 39oC, chills, weakness, hyperaemia,
B. Viral hepatitis type A enlargement, pain and deformity of the
C. Leptospirosis mammary gland. On palpation the infiltrate was
D. Infectious mononucleosis found to have an area of softening and
E. Yersiniosis fluctuation. What is the most likely diagnosis?

80. A 28-year-old woman complains of skin A. Infiltrative-purulent mastitis


hemorrhages after minor traumas and B. Phlegmonous mastitis
spontaneous appearance of hemorrhages on C. Lactostasis
the front of her torso and extremities. On D. Serous mastitis
examination: the skin is variegated (old and E. Mastopathy
new hemorrhages), bleeding gums. Blood
platelets - 20 · 109/L; in the bone marrow there
is increased number of megakaryocytes and no 83. A 10-year-old girl complains of stomachache
platelet production. Treatment with steroid that appears and intensifies after she eats
hormones was effective. What is the likely rough or spicy food, sour eructation, heartburn,
diagnosis? frequent constipations, headaches, irritability.
She has been presenting with these signs for 12 E. Pregnancy I, 39 weeks, pathological
months. Her meals are irregular and consist of preliminary period
dry food. Objectively her diet is suffi- cient in
86. The institutions that take part in medical
calories. The tongue is moist with white coating
examinations include prevention and treatment
near the root. The abdomen is soft and painful
facilities, medical board of Ministry of Defense,
in the epigastrium. What method would be
medical board of Ministry of Home Affairs,
optimal for diagnosis-making in this case?
medico-social expert commissions, forensic
A. Biochemical blood test
medical boards etc. What institutions are
B. Intragastric pH-metry
responsible for temporary disability
C. Fractional gastric analysis (Fractional test
examination?
meals)
A. Medico-social expert commissions
D. Phase-contrast X-ray imaging
B. Sanitary-and-prophylactic institutions
E. Esophagogastroduodenoscopy
C. Prevention and treatment facilities
84. A 64-year-old patient complains of severe D. Medical boards of Ministry of Defense
pain in the right side of chest, dyspnea, dry E. Medical boards of Ministry of Home Affairs
cough which appeared suddenly on exertion.
87. After a lengthy march an army regiment has
Objectively: the right side of the chest lags
set camp for 3 days near a settlement. Sanitary-
behind in the act of breathing. Percussion
hygienic investigation detected several water
reveals tympanic sound. Auscultation reveals
sources. Choose the source that would satisfy
pronouncedly diminished breath sounds on the
the demands for potable water the most under
right. Ps100/min, weak, arrhythmic. AP- 100/50
mm Hg. Cardiac sounds are decreased. What the given field conditions:
disease can be suspected in this patient? A. Melt water B. Brook
C. River
A. Right-sided pneumothorax D. Rain water
B. Right-sided hydrothorax E. Artesian well
C. Right-sided dry pleurisy
D. Right-sided pleuropneumonia 88. Examination of an electric welder with 15
E. PATE years of service record revealed dry rales in the
lower lung fields. Radiograph shows diffuse
85. A 23-year-old primigravida at 39 weeks nodules sized 3-4 mm in the middle and lower
gestation has been admitted to the maternity lung fields. What disease can be suspected?
ward with irregular contractions. The intensity A. Heavy-metal coniosis
of uterine contractions is not changing, the B. Silicosis
intervals between them stay long. Bimanual C. Silicatosis
examination reveals that the cervix is centered, D. Carbon pneumo coniosis
soft, up to 1,5 cm long. There is no cervical E. Bronchitis
dilatation. What diagnosis should be made?
A. Pregnancy I, 39 weeks, labor I, 1 period, the 89. A 30-year-old parturient woman was
latent phase delivered to a maternity hospital with full-term
B. Pregnancy I, 39 weeks, preliminary period pregnancy. She complains of severe lancinating
C.Pregnancy I, 39 weeks, labor I, period 1, the pain in the uterus that started 1 hour ago,
active phase nausea, vomiting, cold sweat. Anamnesis states
D.Pregnancy I, 39 weeks, birth I, 1 period, the cesarean section 2 years ago. Uterine
acceleration phase contractions stopped. Skin and mucous
membranes are pale. Heart rate is 100/min., BP
is 90/60 mm Hg. Uterus has no clear margins, is most probable way of herbicides reaching
sharply painful. No heartbeat can be human organism from soil:
auscultated in the fetus. Moderate bloody A. Soil-animals-human
discharge from the uterus can be observed. B. Soil-microorganisms-human
Uterus cervix is 4 cm open. Presenting part is C. Soil-plants-human
not visible. The most likely diagnosis is D. Soil-protozoa-human
A. Uterine rupture E. Soil-insects-human
B. Initial uterine rupture
93. A 52-year-old patient complains of pain in
C. Threatened uterine rupture
the right part of her chest, dyspnea, cough with
D. Premature detachment of normally
a lot of albuminoid sputum emitting foul smell
positioned placenta
of "meat slops". Objectively: the patient’s
E. Compression of inferior pudendal vein
condition is grave, cyanosis is observed,
90. A parturient woman is 23 years old. Internal breathing rate is 31/min, percussion sound
obstetric examination shows the uterine cervix above the right lung is shortened, auscultation
to be completely open. Fetal bladder is absent. revealed various moist rales (crackles). What is
Cephalic presentation is observed in the plane the most probable diagnosis?
of the small pelvic outlet. Sagittal suture is at A. Chronic pneumonia
the longitudinal section of the small pelvic B. Lung abscess
outlet, small fontanel is situated closer to the C. Pleura empyema
uterus. What cephalic position will the newborn D. Multiple bronchiectasis
have during birth in this case? E. Lung gangrene
A. Longitudinal lie
94. A 15-year-old patient suffers from
B. Minor oblique lie
headache, nasal haemorrhages, sense of lower
C. Transverse lie
extremity coldness. Objectively: muscles of
D. Medium oblique lie
shoulder girdle are developed, lower
E. Major oblique lie
extremities are hypotrophied. Pulsation on the
91. Against the background of angina a patient pedal and femoral arteries is sharply
has developed pain in tubular bones. dampened. BP is 150/90 mm Hg, 90/60 in the
Examination revealed generalized enlargement legs. Systolic murmur can be auscultated above
of lymph nodes, hepatolienal syndrome, carotid arteries. What is the most probable
sternalgia. In blood: RBCs - 3, 6 · 1012/l, Hb87 diagnosis?
g/l, thrombocytes - 45 · 109/l, WBCs - 13 · 109/l, A. Aorta coarctation
blasts - 87%, stab neutrophiles - 1%, segmented B. Aorta aneurism
neutrophiles - 7%, lymphocytes - 5%, ESR - 55 C. Aortal stenosis
mm/h. What is the most likely diagnosis? D. Aortal insufficiency
A. Chronic myeloid leukemia E. Coarctation of pulmonary artery
B. Erythremia
95. A patient with frostbite of both feet was
C. Chronic lymphocytic leukemia
delivered to an admission ward. What actions
D. Acute leukemia
should be taken
E. Multiple myeloma
A. To apply a bandage, to introduce vasodilating
92. For a long time to eradicate weeds on medications
agricultural lands herbicides retaining in B. To administer cardiac medications
environment have been used. Point out the C. To put feet into hot water
D. To rub feet with snow D. Threatened miscarriage
E. To apply an alcohol compress E. Ectopic pregnancy

96. A 26 y.o. male patient with postoperative 99. A 68 year old patient has been suffering
hypothyroidism take thyroxine 100 mg 2 times from chronic pancreatitis for 35 years. During
a day. He has developed tachycardia, sweating, the last 5 years he has been observing
irritability, sleep disorder. Determine further abatement of pain syndrome, abdominal
treatment tactics. swelling, frequent defecations up to 3-4 times a
A. To decrease thyroxine dosage day (feces are greyish, glossy, with admixtures
B. To increase thyroxine dosage of undigested food), progressing weight loss.
C. To administer betablockers Change of symptom set is caused by joining of:
D. To add mercasolil to the treatment A. Chronic enterocolitis
E. To administer sedatives B. Endocrine pancreatic insufficiency
C. Syndrome of lactase deficiency
97. A 20 y.o. patient was admitted to the D. Irritable bowels syndrome
hospital with complaints of having skin and E. Exocrine pancreatic insufficiency
sclera icteritiousness, dark urine, single
vomiting, appetite loss, body temperature rise 100. A patient suffering from acute
up to 380 for 2 days. Three weeks ago he went posttraumatic pain received an injection of
in for fishing and shared his dishes with friends. morphine that brought him a significant relief.
Objectively: the patient is flabby, t 0- 36, 80, Which of the following mechanisms of action
skin and scleras are icteritious, liver sticks from provided antishock effect of morphine in this
under the costal margin by 3 cm, it is sensitive; patient?
spleen isn’t palpable. Urine is dark, stool is A. Block of central cholinergic receptors
partly acholic. What is the most probable B. Stimulation of opiate receptors
diagnosis? C. Stimulation of benzodiazepine receptors
A. Infectious mononucleosis D. Inhibition of dopamine mediation
B. Leptospirosis E. Intensification of GABA-ergic reactions
C. Virus A hepatitis
101. During the induction of labor of a 41.5-
D. Hemolytic anemia
week multigravida woman, with misoprostol,
E. Intestinal yersiniosis
the mother begins to feel increasing pelvic
98. A 36-year-old female pesented to a pressure and low back pain. FHTs are reassuring
gynecological hospital with a significant bleeding and maternal vital signs are p-92, BP 110/68, R-
from the genital tract and a 1-month delay of 18; T-37 contractions are every 2 to 3 minutes
menstruation. Bimanual examination revealed and moderate. Her cervix is 2 cm50%, —2
soft barrel-shaped cervix. Uterus was of normal station with blood tinged mucus. What does
size, somewhat softened. Appendages were this situation represent?
unremarkable on both sides. Speculum (A) impending rupture of the uterus
examination revealed that the cervix was (B) normal early labor
cyanotic, enlarged, with the the external orifice (C) tachysystole
disclosed up to 0,5 cm. Urine hCG test was (D) misoprostol syndrome
positive. What is the most likely diagnosis? (E) primary arrest of labor
A. Uterogestation 102. A 23-year-old woman with right-sided
B. Cervical pregnancy lower abdominal pain and chills is seen in the
C. Abortion in progress
emergency department. The pain began 3 days other complaints recorded. Anamnesis states
ago and is associated with a vaginal discharge. causeless urticaria and skin itching that
Her LMP was 5 days ago. She uses an occurred 1 year ago. Objectively: liver margin is
intrauterine device for contraception and had rounded, painless, and can be palpated 3 cm
coitus 1 week ago with her new boyfriend. below the costal arch. No other pathology was
There is no history of nausea, vomiting, or detected during physical examination of the
diarrhea. Her vital signs are blood pressure, patient. Body temperature is normal. X-ray
120/80 mmHg; pulse, 100 bpm; and reveals a hemispherical protrusion in the right
temperature, 101.4°F. Abdominal examination cupula of the diaphragm. What disease can be
shows bilateral lower quadrant guarding with suspected in the given case?
rebound tenderness on the right side. Pelvic A. Liver abscess
examination shows pus at the cervical os and a B. Hydatid disease of liver
tender 6-cmright adnexal mass. Laboratory data C. Hepatocellular carcinoma
are hematocrit, 38% (normal, 35% to 45%); D. Metastatic tumor
WBC count, 25,000/mL (normal, 3–10,000/mL); E. Subdiaphragmatic abscess
and serumpregnancy test, negative.
105. A 33-year-old woman presents with a
Transvaginal sonography shows a 6-cmcomplex
cellulitis of the dorsum of the right hand 24
right adnexal mass. The uterus and left adnexa
hours following a cat bite. A Gram stain of
are normal. Which of the following is the most
serosanguinous discharge from a puncture site
likely diagnosis?
shows neutrophils and small gram negative
(A) appendicitis
rods. What is the treatment of choice for this
(B) adnexal torsion
patient?
(C) pyosalpinx
A. cefazolin
(D) hydrosalpinx
B . norfloxacin C. amoxicillin/clavulanic acid
(E) endometritis
D. gentamicin
103. A 36-year-old man has had stable
106. In utero heart failure, often with fetal
ulcerative colitis for 5 years. He is being treated
pleural and pericardial effusions and
with sulfasalazine 1000 mg three times daily.
generalized ascites (non-immune hydrops
On routine follow-up, he is found to have an
fetalis) may occur in ?
ALT of 250 IU/L, an AST of 225 IU/L, an alkaline
A. ventricular septal defect
phosphatase of 450 IU/L, and a bilirubin of 2
B. coarctation of aorta
μmol/L. Ultrasound of the abdomen reveals no
C. d-Transposition of great arteries
bile duct dilatation or gallstones. Endoscopic
D. Ebstein anomaly
retrograde cholangiopancreatography (ERCP)
E. single ventricle
reveals multifocal strictures and dilatations of
both the intrahepatic and extrahepatic bile 107. You are discussing the risk of radiotherapy
ducts. What is the MOST likely diagnosis? with the parents of a child with
A. sclerosing cholangitis medulloblastoma; the mother has a concern
B. common bile duct stone about the late neurological complications post
C. primary biliary cirrhosis radiotherapy. The statement that should be
D . drug-induced hepatitis included in the discussion that late neurological
sequelae post radiotherapy is more severe
104. A 41-year-old patient, a hunter, complains
with?
of heaviness in the right subcostal area. No
A. focal radiotherapy rather than craniospinal
irradiation attacks in this disease?
B. children with an age of less than 3 year A. Erythrocytic schizogony
C. concomitant chemo-radiotherapy B. Tissue schizogony
D. low grade tumors rather than high grade C. Exotoxin of a causative agent
tumors D. Endotoxin of a causative agent
E. infratentorial tumors rather than E. Gametocytes
supratentorial tumors
111. A patient is 30 years old, works as a
108. Which of the following statements are carpenter. Six months ago there appeared some
true? behavioural changes: he got interested in
A. Identification of the causative organism philosophy, began writing a treatise on the
should be done before starting antibiotics. purpose of his human existence, quitted his job,
B Wounds are best managed by delayed stopped caring about his children, went out
primary or secondary closure. carelessly dressed, heard "voices in his
C Subcuticular continuous skin closure head"that guided his behaviour. The patient
decreases the incidence of wound infection. claimed sure that he was an Ambassador of God
D Polymeric films can be useful in infected on Earth and was constantly feeling His
wounds. influence. He is not critical about his disease.
E Administration of antibiotic preparations What diagnosis can be assumed?
locally is more effective than the oral route. A. Reactive psychosis
B. Alcocholic psychosis
109. Which of the following statements
C. Schizophrenia
regarding burn depth are true?
D. Somatogenic psychosis
A) The depth of a burn together with
E. Organic psychosis
percentage of TBSA and smoke inhalation are
key parameters in the assessment and 112. A 27-year-old patient complains of nasal
management of a burn. haemorrhages, multiple bruises on the anterior
B)Alkalis, including cement, usually result in surface of the trunk and extremities, sudden
superficial burns. weakness. In blood: Hb- 74 g/l, reticulocytes -
C) Fat burns are deeper than electrical contact 16%, RBCs - 2, 5 · 1012/l, platelets - 30 · 109/l,
burns. ESR- 25 mm/h. What is the most effective
D )Capillary filling is not present in superficial measure for the treatment of
burns. thrombocytopenia?
E) Deep dermal burns take a maximum of 2 A. Iron preparations
weeks to heal without surgery. B. Splenectomy
C. Hemotransfusion
110. A 47-year-old patient came to see a doctor
D. Cytostatics
on the 7th day of disease. The disease
E. Vitamin B12
developed very fast: after the chill body
temperature rose to 40oC and lasted up to 7 113. 2 days ago a patient presented with acute
hours, then dropped abruptly, which caused pain in the left half of chest, general weakness,
profuse sweat. There were three such attacks fever and headache. Objectively: between the 4
occuringonce in two days. Two days ago the and 5 rib on the left the skin is erythematous,
patient arrived from Africa. Objectively: pale there are multiple groups of vesicles 2-4 mm in
skin, subicteric sclera, significantly enlarged diameter filled with transparent liquid. What
liver and spleen. What is the cause of fever diease are these symptoms typical for?
A. . Streptococcal impetigo 117.A healthy child 1 year and 5 month of age is
B. Pemphigus being vaccinated agains the hapatitis B. The child
C. Herpes simplex did not receive the first dose the vaccine
D. Herpes zoster previously, while in the maternity hospital. The
E. Herpetiform Duhring’s dermatosis doctor makes an individual vaccination schedule
for this child and planning the administration of
114. After lifting a load a patient felt undurable
the next dose the vaccine. What is the minimum
pain in the loin. He was diagnosed with acute
interval between doses of vaccine in this case?
lumbosacral radiculitis. Which of the following
is contraindicated for this patient? A. 2 months B. 3 months C. 6 months D. 12
A. Warming procedures months E. 1 month
B. Dehydrating drugs
118. A 19-year-old has been bleeding
C. Analgetics
intermittently for 26 weeks. She presents with a
D. Vitamins of B group
change to dark bleeding and minimal cramping
E. Intravenous injection of aminophylline
for 4 days. Your clinic ultrasound notes an
115. A 30-year-old woman complains of intrauterine fetal demise. She is distraught. She
irregular copious painful menstruations, pain would prefer to let “nature take its course.” She
irradiates to the rectum. Anamnesis states asks if there is any risk to her. What do you
10year-long infertility. On bimanual explain as the only significant risk?
examination: uterus is of normal size; uterine (A) a positive human chorionic gonadotropin
appendages on the both sides are corded, with (hCG) titer (B) systemic allergies (C) bone
rectricted mobility, painful; there are dense marrow depression (D) coagulopathy (E)
nodular painful growths detected in the toxemia
posterior fornix. A doctor suspects
endometriosis. What method allows to verify 119. A 15-year-old patient complains of
this diagnosis? excessive body weight, headache, irritability,
rapid fatigability. Significant increase of body
A. Laparoscopy weight occurred at the age of 14. Objectively:
B. Diagnostic curettage of uterine cavity
weight is 90 kg; height is 160 sm, proportional
C. Paracentesis of posterior fornix body built. Fatty tissue is distributed evenly.
D. Uterine probing There are thin pink striae (stretch marks) on the
E. Hysteroscopy
thighs, abdomen and mammary glands. BP -
116.A 60-year-old man presents with 145/90 mm Hg. Provisional diagnosis is:
subcompensated viral liver cirrhosis (HCV Child- A. Pubertate dyspituitarism
Pugh class B. What tactics should I chosen B. Alimentary constitutive obesity
regarding the vaccination again influenza in this C. Somatoform autonomic dysfunction
case? D. Itsenko-Cushing’s disease
A. . Contraindicated due to disease progress E. Cushing’s syndrome
stage, as shown by Child- Pugh class
B. Scheduled yearly vaccination 120.A 12-year-old girl after a case of respiratory
C. In case of influenza outbreak infection developed dyspnea at rest, paleness of
D. Combined with antiviral drugs skin. Heart rate is 110/min., BP is 90/55 mm Hg.
E. Contraindicated due to elderly age of patient Heart sounds are muffled. Borders of relative
heart dullness: right - the parasternal line, upper
- the III rib, left - 1,0 cm outwards from the
midclavicular line. Make the provisional
diagnosis: 124. Describe the coronavirus structure.
A. Somatoform autonomic dysfunction A .Club shaped glycoprotein spikes protrude
B. Functional cardio C. Infectious myocarditis through a lipid bilayer
D.Hypertrophic cardiomyopathy E. Exudative B. An icosahedral structure with an envelope
pericarditis C. An icosahedral large pleomorphic virus
D. Large regimented barrel shaped virus
121.A child is 9 months old. The patient’s body
temperature is 36,7oC, the skin is pale, humid, 125. A 21-year-old male, working in a grocery
there is pain in leg muscles. There is no store in California where the virus has spread,
extremities mobility, sensitivity is present. The had high fever, fatigue, and body ache similar to
child has been diagnosed with poliomyelitis. the symptoms of regular flu/common cold. He
The causative agent of this disease relates to was suspected of COVID-19 by his doctor. He
the following family: had mild symptoms and was recommended to
isolate and take care of himself at home. If the
A. Paramyxovirus
symptoms worsen with difficulty in breathing, a
B. Picornavirus
confirmation test for COVID-19 must be
C. Tohovirus
performed. All of the following statements are
D. Adenovirus
true for the COVID-19 test, EXCEPT?
E . Rotavirus
A. Nasal swab, throat swab, and saliva are taken
122. A parturient woman is 27 year old, it was as a sample for the detection of the virus
her second labour, delivery was at term, normal B. The samples taken immediately after the
course. On the 3rd day of postpartum period exposure to the virus may give false-negative
body temperature is 36,8oC, Ps - 72/min, AP - test results
120/80 mm Hg. Mammary glands are C. A blood test is the primary diagnostic test for
moderately swollen, nipples are clean. the identification of the virus
Abdomen is soft and painless. Fundus of uterus D. RT-PCR technique is used as a diagnostic test
is 3 fingers below the umbilicus. Lochia are for the identification of the virus
bloody, moderate. What is the most probable
diagnosis? 126 A 38-year-old African American woman
presents with heavy menses and an enlarged
A. Physiological course of postpartum period uterus. After an examination the clinical
B. Subinvolution of uterus diagnosis is leiomyoma of the uterus. Which of
C. Postpartum metroendometritis the following best describes this finding?
D. Remnants of placental tissue after labour (A) a soft, interdigitating mass of the uterine
E . Lactostasis wall
(B) a premalignant papule of the uterine wall
123 Which of the following is an unusual (C) a rapidly dividing necrotic malignancy
feature of the replication cycle in (D) a rounded, smooth, firm, well-circumscribed
coronaviruses? mass
A. They take advantage of recombination with (E) erythematous, tender, and hereditary
the long RNA genome
B. The RNAs all terminate in a common 3´ and 127. A 16-year-old girl is brought to clinic by her
produce nested set transcripts. mother with concerns regarding the lack of any
C.They are not highly mutable. signs of puberty. The appearance of external
D.They use capped cellular mRNA's. genitalia is of a normal prepubertal female.
Laboratory studies show markedly elevated FSH following, the MOST appropriate
and LH levels. Which of the following causes of explanation for the child behavior is
delayed puberty accompanies elevated
circulating gonadotropin levels? A) acute phobic hallucination B) early sign
(A) chronic illness of schizophrenia C)delusional infestation
(B) gonadal dysgenesis D) night terror E)bipolar disorder
(C) hypothalamic tumors
(D) Kallmann syndrome
(E) malnutrition

128. A 65-year-old man who has lived all his life


in Britain presents with severe and constant
back pain at L3, which does not radiate down
the legs. It wakes him at night and he has night
sweats. X-rays show destruction of the disc
space at L2/L3 with invasion of the destruction
into the adjacent vertebral bodies. What would
you expect to find on needle biopsy?
A Tumour cells (metastases)
B Primary chondrosarcoma cells
C Mycobacterium tuberculosis
D Escherichia coli
E Sterile avascular necrosis.

129. A 35-year-old multiparous woman presents


with marked dyspnea and orthopnea in the
ninth month of what is known to be a twin
pregnancy. She is in atrial fibrillation, and
her blood pressure is 130/95 mm Hg. She
has elevated jugular venous pressure,
cardiomegaly, and a third heart sound.
There is a grade 2/6 pansystolic murmur at
the apex. Chest examination reveals
bibasilar inspiratory crackles. In addition to
digoxin and furosemide, which of the
following medications would be MOST
appropriate?
A. hydralazine B. captopril C. prednisone
D. warfarin

130 A parents came to your clinic complaining


that their 5-year-old boy had attacks of
frightening with imagination of a snake
crawling over him and he is acting as trying
to remove it. You assessed the child and
find no acute physical illness and he is
cooperative and quite intelligent. Of the
Dr. Ashiq’s Tutorials

PreKrok Krok-2
Medicine
Online Exam
Test timing- 2:40Hrs

This is for your self-evaluation purpose, Don’t cheat yourself

Name:
Whatsapp no:
University:

“Make your parents proud


your enemies jealous &
yourself happy”

+91 9207609873
1. Clinical trials have patients with dysentery.
proved the ”Lipoflavon” What accounting
drug to be effective for document reflects this
treatment of unstable type of morbidity?
angina pectoris in the A. Statistical report
control group and B. Urgent report
C. Report on a major non-
experimental group of epidemic disease
patients. Neither patients D. Certificate of temporary
disability
nor researchers knew who E. Control card of a patient
belonged to which group. registered for regular
Name this type of study: check-ups
A. Total-blind study
B. Simple blind study
C. Triple-blind study
D. Double blind study 4. During regular
E. Multicenter study preventive gynecological
examination a 30-year-old
woman was detected to
2. In a rural health care have dark blue punctulated
area there is an increasing ”perforations” on the
cervical cancer morbidity vaginal portion of the
observed. The decision is uterine cervix. The doctor
made to conduct a suspects endometriosis of
medical examination of the the vaginal portion of the
women living in this locality. uterine cervix. What
What type of medical investigation method
examination is it? would be most informative
A. Target for diagnosis confirmation?
B. Preliminary
C. Regular A. Colposcopy, target biopsy
D. Complex of the cervix
E. Screening B. US of the lesser pelvis
C. Hysteroscopy
D. Curettage of the uterine
cavity
3. A district doctor has E. Hormone testing
diagnosed one of his
5. A 22-year-old post maternity hospital. The doctor
parturient woman on the makes an individual vaccination
12th day after the
normal childbirth informs schedule for this child and
of elevated body plans the administration of the
temperature up to 39oC next dose of the vaccine. What
for the last 3 days and is the minimum interval
pain in her right between doses of vaccine in
mammary gland. The
right mammary gland is this case?
enlarged, hot to touch, A. 3 months B. 6 months
tense, hyperemic, and C. 2 months D. 1 month
painful. Palpation reveals E. 12 months
there a dense infiltration
8x8 cm with a fluctuation
in its center. What is the
most likely diagnosis? 7. The mother of an epileptic
son complains of her son
A. Postpartum period, day
12. Right-sided gangrenous periodically presenting with
mastitis irritable depression, when he
B. Postpartum period, day becomes excited and prone to
12. Right-sided serous physical aggression and
mastitis
C. Postpartum period, day
violence. These moods last for
12. Right-sided infiltrative- 5-10minutes, after which the
purulent mastitis patient exhausts himself and
D. Postpartum period, day falls asleep. Upon waking he is
12. Right-sided
phlegmonous mastitis depressed, sad, cannot recall
E. Postpartum period, day his actions or recalls the monly
12. Right-sided lactostasis partially. What
psychopathologic condition is it
6. A healthy child 1 year and 5 ?
months of age is being A. Ambulatory automatism
vaccinated against hepatitis B. B. Dysthymia
The child did not receive the C. Epileptic delirium
first dose of the vaccine D. Pathologic affect
previously, while in the E. Dysphoria
8. During examination a 4- emergency teams
month-old child with E. In medical institutions;
meningococcemia presents all listed types of
with acrocyanosis, cold healthcare workers
extremities, tachypnea,
and thready pulse, blood
pressure of 30/0 mm Hg, 10. A boy had a foreign body
anuria, and sopor. What removed from under his nail
clinical syndrome is it? plate. 3 days later he
developed a sharp
A. Neurotoxicosis
B. Toxic shock syndrome throbbing pain at the end of
C. Exicosis his distal phalanx, which
D. Encephalic syndrome
E. Acute renal failure intensifies when the phalanx
is pressed, hyperemia of the
9. At the railroad crossing a nail fold, elevated body
passenger train collided
with a bus. In this collision temperature up to 38.5o C,
and nail plate discoloration.
26 bus passenges died,
Make the diagnosis:
another 18 passengers
received mechanical A. Erysipeloid
B. Erysipelas
injuries of varying severity. C. Paronychia
Where will be professional D. Subungual panaritium
E. Abscess
medical aid provided for
the victims of this
accident? Who will provide 11. A 45-year-old man
this aid? diagnosed with acute
pulmonary abscess
A. At the site of the
accident; specialized suddenly developed sharp
second-response pain in his chest on the
emergency teams right and dyspnea up to
B. At the site of the
30/min. Examination
accident; first-response detects facial cyanosis and
emergency teams shallow rapid respirations.
C. In medico-prophylactic
Auscultation reveals
institutions; general acutely weakened
physicians and surgeons respiration throughout the
D. In medico-
whole right lung;
prophylactic percussion reveals a
institutions; specialized vesiculotympanitic
second-response (bandbox) resonance at the
lung apex and dullness in
the lower lobe. What A. Tetanus
B. Anaerobic gas gangrene
complication developed in C. Erysipelas
this patient? D. Acute thrombophlebitis
E. Thrombophlebitis of the
A. Pyopneumothorax popliteal artery
B. Pleuropneumonia
C. Pneumothorax
D. Acute mediastinitis
E. Esophageal perforation 14. A 48-year-old woman
has arrived to the surgical
unit with wounds in her
12. The body of a 24-year- thigh. On examination the
old woman with suspected wound surface has dirty-
poisoning has been found gray coating with
on the street. Forensic unpleasant sweet smell.
medical examination was Wound content resembles
requested by an raspberry jelly. Skin tissues
investigator during around the wound are
examination of the site and glossy and turgid. Palpation
the body. According to the reveals moderate
Criminal Procedure Code crepitation in the tissues.
currently in force in What microflora is the
Ukraine, forensic medical most likely to cause such
examination is required inflammation?
when it is necessary to A. Streptococci
determine the: B. Anaerobic non-clostridial
C. Anaerobic clostridial
A. Mode of death D. Staphylococci
B. Manner of death E. Blue pus bacillus
C. Time of death
D. Cause of death
E. Mechanism of death
15. A 65-year-old woman
on abdominal palpation
13. On the 15th day after a presents with a tumor in
small trauma of the right the umbilical region and
foot, the patient developed above it; the tumor is 13x8
indisposition, fatigability, cm in size, moderately
irritability, headache, painful, non- mobile,
elevated body temperature, pulsing. On auscultation
and sensation of systolic murmur can be
constriction, tension, and observed. What is the most
twitching in the muscles of likely diagnosis?
the right shin. What disease
A. Arteriovenous aneurysm
can be suspected? B. Gastric tumor
C. Abdominal aortic fingers, hands, wrists,
aneurysm
D. Tricuspid insufficiency elbows, lower abdomen,
E. Bicuspid insufficiency genitals, and thighs there
are paired papulovesicles,
16. A 26-year-old man single pustules, and
complains of chills, scratch marks. What
rhinitis, dry cough, and disease can be suspected?
fever up to 38oC. A. Shingles
B. Pyoderma
Examination shows him to C. Dermatitis
be in a moderately severe D. Eczema
E. Scabies
condition; there are small
pale pink non-merging
spots on the skin of his 18. A 48-year-old woman
back, abdomen, and has been hospitalized due
extremities. Palpation to development of
reveals enlarged occipital tachysystolic atrial
and fibrillation. She has lost 5
axillary lymph nodes. No kg of body weight within 2
information about months. On palpation
vaccination history could there is a node in the left
be obtained. What is the lobe of the thyroid gland.
likely etiology of this What pathology resulted
disease? in the development of this
A. Mumps virus condition?
B. Epstein-Barr virus
C. Streptococcus A. Toxic nodular goiter
D. Rubella virus B. Aterosclerotic
E. Neisseria meningitis cardiosclerosis
C. Chronic thyroiditis
D. Nontoxic nodular goiter
E. Autoimmune thyroiditis
17. The dermatologist has
an appointment with a 19. A 72-year-old man
30-year-old man that diagnosed with ischemic
heart disease presents with
complains of severely diffuse cardiosclerosis,
itching rashes that permanent tachysystolic
especially disturb him at atrial fibrillation, heart
failure IIа, FC III. Objective
night. The rashes examination of vital signs:
developed 2 weeks ago, blood pressure is 135/80
mm Hg, heart rate is
after he had returned from 160/min., pulse is 125/min.
a travel. Objectively on the Left ventricular ejection
fraction is 32%. What drug
lateral surfaces of his is indicated in this case and
should be presribed to the the patient fell asleep but
patient?
in 5 minutes the second
A. Procainamide
(Novocainamide) attack occurred. The first
B. Digoxin step of emergency aid
C. Isadrine (Isoprenaline) would be to:
D. Verapamil
E. Ivabradine A. Prescribe antiepileptic
drugs
B. Take blood from the vein
for analysis
20. A 39-year-old man C. Introduce diazepam
suffers from chronic intravenously
D. Ensure patency of airways
rheumatic heart disease. E. Administer chloral
He complains of dyspnea hydrate via an enema
during physical exertion,
cough with expectoration, 22. A 45-year-old man, a
and palpitations. farmer, presents with acute
Ausculation detects onset of a disease. He
intensified I heart sound complains of headache,
and diastolic murmur; the high temperature, pain in
sound of opening mitral the gastrocnemius muscles,
valve can be auscultated icteric face, and dark urine.
at the cardiac apex. The Objectively: body
II heart sound is
temperature - 38oC, blood
accentuated over the pressure - 100/70 mm Hg,
pulmonary artery. The conjunctival hemorrhages,
patient is cyanotic. X-ray hepatosplenomegaly, and
shows dilated pulmonary oliguria. What is the most
root and enlargement of likely provisional
the right ventricle and left diagnosis?
atrium. What is the most
A. Viral hepatitis
likely diagnosis? B. Brucellosis
C. Leptospirosis
A. Pulmonary artery stenosis D. Pseudotuberculosis
B. Aortic stenosis Trichinosis
C. Mitral stenosis
D. Coarctation of the aorta
E. Patent ductus arteriosus

23. A 20-year-old student


21. A 35-year-old patient after failing an exam
developed an epileptic developed complaints of a
attack with tonoclonic sensation of a round
spasms that lasted for 3 foreign body in her throat,
minutes. After the attack difficult swallowing. She
fixates on her condition, Temperature is 36.6oC.
limits her diet, often cries, Gregersen reaction(occult
seeks attention, exhibits
demonstrative attitude. blood in feces)is negative. Stool
She is highly susceptible to culture detected no pathogenic
psychotherapeutic germs. What is the most likely
suggestion. What cause of diarrhea in this case?
psychiatric diagnosis can
be made in this case? A. Ulcerative colitis
A. Hysterical neurosis B. Intestinal dysbiosis
B. Hypochondriacal neurosis
C. Depressive neurosis C. Bacterial over growth
D. Obsessive neurosis
E. Paranoid personality syndrome
disorder
D. Antibiotic-associated
diarrhea
24. A 42-year-old man, a
dispatcher, suffers from E. Crohn’s disease (regional
peptic ulcer disease of the enteritis)
duodenum. The disease is
of moderate severity. He 26. Caries morbidity rate is 89%
wants to be assigned a among residents of a
disability group. Make the community. It is determined
conclusion regarding his
working ability: that fluorine content in water is
A. First group of disability
0.1mg/L.What preventive
B. Capable of working, non- measures should be taken?
employable
C. Capable of working,
employable A. Introduce more vegetables
D. Second group of disability
E. Third group of disability to the diet
B. Tooth brushing
25. A 34-year-old man on the C. Fluorine inhalations
3rd day of ceftriaxone D. Seal ant application
treatment for acute otitis (daily E. Water fluorination
dosage - 2grams) developed 27. A patient has gradually lost
diarrhea occurring 5-6times per consciousness. The skin is pale
day. Feces are with out mucus and dry. Theres a smell of
or blood ad mixtures. ammonia from the mouth.
Respirations are deep and reappeared fibrin film.Protein-
noisy. Heart sounds are 1.4g/L,lymphocytes-600/3 per
muffled, pericardial friction rub mm3, sugar-0.3mmol/L. What
is present. Blood pressure is is the provisional diagnosis?
180/130mm Hg. Blood test:Нb- A. Meningococcal meningitis
80g/L, leukocytes- 12 · 109/L,
B. Tuberculous meningitis
bloodglucose-6.4mmol/L, urea-
C.Lymphocytic(Armstrong’s)
50mmol/L,creatinine- meningitis
1200mcmol/L, blood osmolarity
D. Syphilitic meningitis
350mOsmol/L. No urinary E. Mumps meningitis
excretion. Make the diagnosis:
29. A 16-year-old girl has
A. Acute renal failure primary amenorrhea, no
B. Hyper glycemic coma pubic hair growth, normally
C. Uremic coma developed mammary
glands; her genotype is 46
D. Acute disturbance of
ХY; uterus and vagina are
cerebral circulation absent. What is your
E. Hyperosmolarcoma diagnosis?
28. A 20-year-old patient A. Sheehan syndrome
complains of severe headache, B. Mayer-Rokitansky-
double vision, weakness, fever, Kuster-Hauser syndrome
C. Cushing syndrome
irritability. Objectively: body D. Testicular feminization
syndrome
temperature is 38.1oC, the E. Cushing disease

patient is reluctant to contact,


30. A 13-year-old girl has
sensitive to stimuli. There are 30% of excessive body
ptosis of the left eyelid, mass, she started to gain
exotropia, anisocoria S>D, weight at the age of 3. She
pronounced meningeal has a family history of
syndrome. On lumbar puncture obesity. Her height and
sexual development are
the cerebrospinal fluid flowed normal for her age. The
out under a pressure of 300mm appetite is excessive. She
Hg, the fluid is clear, slightly complains of periodical
opalescent. 24 hours later the headaches. Blood pressure
- 120/80 mm Hg.
Subcutaneous fat is evenly D. Prostaglandin E1 therapy
distributed, she has no E. Reassurance, no further
stretch marks. There is
juvenile acne on her face. workup
What type of obesity is it? F. Surgical repair
A. Hypothalamic obesity
B. Alimentary constitutive
obesity 32. A 7-year-old girl is brought
C. Adrenal obesity to the physician for a routine
D. Hypothalamic syndrome
of puberty physical examination. She and
E. Hypothyroid obesity
her parents have no
complaints. The girl is in the
second grade and has several
31. A 1-month-old boy is friends at school. She can tie
brought to the physician for a her shoes independently and
routine visit. His prenatal and ride a 2-wheel bicycle. Weight
birth histories are is at the 50th percentile and
unremarkable. His vital signs height is at the 5th percentile.
and pulse oximetry are normal. Examination shows a short,
Examination shows a harsh, webbed neck; a broad chest;
III/VI holo systolic murmur over and widely spaced nipples.
the left lower sternal border. Karyotype results are
Palpation Indicates no thrill abnormal. Which
over the precordial region. cardiovascular anomaly is most
Brachial and femoral pulses are likely to be present in this
2+ and equal. The skin is pink patient?
and well perfused.
A. Atrial septal defect
Electrocardiogram is normal.
B. Bicuspid aortic valve
Which of the following is the
C. Mitral valve prolapse
most appropriate course of
D. Patent ductus arteriosus
action?
E. Tetralogy of Fallot
A. Digoxin and diuretic therapy F. Ventricular septal defect
B. Echocardiography
C. Oxygen administration
33. A 15-year-old boy collapses difficulty in adapting to
suddenly while playing darkness. He is a very poor
basketball with his friends. The eater, and his diet consists
boy played basketball every mainly of canned foods, and
weekend without any very rarely, fresh vegetables or
symptoms and was in his usual milk. Examination reveals dry,
state of health prior to this scaly skin, follicular
event. He has no known past hyperkeratosis in the extensor
medical history and takes no surfaces of the extremities, and
medications. Family history is dry, silver-gray plaques on the
significant for a maternal uncle bulbar conjunctiva. What is the
who died at age 25 for most likely diagnosis of this
unknown reasons. patient?
Cardiopulmonary resuscitation
A. Vitamin A deficiency
is unsuccessful and the patient
B. Thiamine deficiency
is pronounced dead. Post- C. ribo flavinosis
mortem examination would
D. Scurvy
most likely show which of the
E. Hyper vitaminosis A
following?
A. Abnormal brain mass
35. An 8-year-old African
B. Coronary atherosclerosis American child is brought to
C. Hypertrophic
the office for the evaluation of
cardiomyopathy
a 1-day history of fever and
D. Mutation in the LQT1 gene back pain. He has sickle cell
E. No underlying disease
disease, and has had 5
F. Pulmonary embolus hospitalizations for similar
G. Ruptured aortic aneurysm
painful crises. His laboratory
report shows normocytic
34. A 3-year-old boy is brought anemia, reticulocytosis and
to the office by his parents for
leukocytosis. What finding is
the evaluation of dry eyes and
most likely to be present on
photophobia. He has some this patient's peripheral smear?
A. Bite cells to worsen this patient's eye
B. Helmet cells disease?
C. Howell Joly bodies
A. Prednisone
D. Heinz bodies
B. Propranolol
E. Basophilic stippling C. Propylthiouracil
D. Radioactive iodine
36. A 58-year-old woman E. Thyroidectomy
comes to the office due to
recent-onset tremor, 37. A 25-year-old woman
palpitations, weight loss, and
comes to the physician with a
fatigue. She has no other
3-month history of weight loss.
medical conditions and takes Irritability, Insomnia, and
no medications or over-the-
palpitations. Her past medical
counter supplements. The
history is Insignificant. She is
patient's mother had not currently taking any
hypothyroidism and
medications. The patient does
osteoporosis. Blood pressure is
not use tobacco, alcohol, or
144/80 mm Hg. pulse is illicit drugs. Blood pressure is
110/min, and respirations are
155/70 mm Hg and pulse is
18/min. On examination, there
110/min. Physical examination
is diffuse, nontender shows lid retraction, dry skin,
enlargement of the thyroid
and fine tremor of the hands.
gland. Ocular examination
Which of the following is the
shows bilateral proptosis, lid most Iikely cause of this
lag, and periorbital puffiness.
patient's hypertension?
The patient has diplopia and
ocular discomfort on extremes A. Elevated serum calcium level
of lateral gaze. Laboratory tests B. Increased myocardial
show a suppressed TSH and contractility
elevated thyroid homone C. Increased peripheral vascular
levels. Which of the following resistance
treatment options is most likely D. Increased synthesis of
catecholamines
E. Nocturnal hypoventilation right eye photophobia, and
excessive lacrimation Gross
examination of the right eye
38. A 39-year-old woman who
with a penlight after the
recently underwent
thyroldectomy for Grave's application of a topical
anesthetic is insignificant. What
disease complains of anxiety,
is the best next step in the
muscle cramps, and poor sleep.
Her medications include daily management of this patient?
multivitamin supplements and A. Tonometry
lansoprazole for acid reflux B Fluorescein examination
disease. Her temperature is C. Topical antibiotic
36.6 C (98 F). blood pressure is D. Ultrasoragraphy
142/85 mm Hg, and pulse is E. MRI of the orbits
82/min. Electrocardiogram
shows normal sinus rhythm 40. A 65-year-old man comes to
with prolongation of the
the emergency department due
corrected QT interval to 510 m
to a sudden loss of vision in his
sec (normal <460 m sec). Which right eye. He has had diabetes,
of the following is the most
and has been treated with
likely diagnosis?
metformin and glyburide for
A. Hyperkalemia the past 10 years. visual acuity
B. Hypocalcemia is reduced to light perception in
C. Hyponatremia his right eye, and normal in his
D. Persistent hyperthyroidism left. His vital signs are normal.
E. Vitamin D toxicity Ophthalmoscopy reveals loss of
fundus details, floating debris
39. A 30-year-old man comes to and a dark red glow. What is
the emergency department the most likely diagnosis?
screaming, "Something blew
A. Retinal detachment
into my right eye while I was
B. Diabetic retinopathy
drilling!" He complains of a C. Vitreous hemorrhage
foreign body sensation in the
D. Central retinal vein occlusion
E. Age related macular content. What is the most likely
degeneration diagnosis?
A. Pemphigus
41. During assessment of work B. Herpes simplex
conditions at the factory C. Streptococcal impetigo
manufacturing mercury D. Herpes zoster
thermometers, the content of E. Dermatitis herpetiformis
mercury vapors in the air of the (Duhring’sdisease)
working area is revealed to 43. A prematurely born girl is
exceed the maximum now 8
concentration limit. Specify the months old. She has dyspnea,
main pathway of human body tachycardia,
exposure to mercury: hepatosplenomegaly, physical
A. Damaged skin developmental lag, limb
cyanosis. There is parasternal
B. Intact skin
cardiac hump, auscultation
C. Respiratory organs
revealed systolodiastolic
D. Gastrointestinal tract murmur in the II intercostal
E. Mucous tunics space on the left. BP is 90/0
mm Hg. What disease can be
suspected?
42. The doctor has an A. Stenosis of pulmonary artery
appointment with a patient, B. Coarctation of aorta
who 2 days ago developed C. Stenosis of aortal valve
severe chest pain on the left, D. Patent ductus arteriosus
general weakness, high E. Nonclosure of
temperature, and headache. interventricular septum
Objectively along the 4th and 44. A worker of a glass-blowing
5th intercostal nerves on the workshop complains of
left the skin is hyperemic and headache, irritability, visual
there are tight clusters of small impairment - he sees
everything as if through a
vesicles filled with clear serous
”net”. Objectively: hyperemic
sclera, thickened cornea,
decreased opacity of pupils, 46. A full-term new born (born
visual acuity is 0,8 in the left with the body weight of 3900 g
eye, 0,7 in the right eye. The at gestational age of 39 weeks)
worker uses no means of on the first day of his life
personal protection. What is developed respiratory
the most likely diagnosis? disturbances: dyspnea,
A. Cataract arrhythmic respiration,
B. Conjunctivitis cyanosis attacks. On
C. Keratitis examination there is
D. Blepharospasm paradoxical respiration
E. Progressive myopia observed and left side of the
chest lags behind in the act of
breathing. On auscultation the
45. An 9-year-old child was respiration is weakened in the
hospitalized for fever up to lungs on the left. Neurologist
39,8oC, inertness, moderate diagnosed the patient with left-
headache, vomiting. sided ErbDuchenne palsy.
Examination revealed Complete blood count shows
meningeal symptoms. Lumbar no changes.What is the most
puncture was performed. The likely diagnosis?
obtained fluid was A. Respiratory distress
characterised by increased syndrome
opening pressure, was B. Congenital pneumonia
transparent, with the cell count C. Left-sided pneumothorax
of 450 cells per 1 mcL (mainly D. Left-sided diaphragm paresis
lymphocytes - 90%), glucose E. Transient tachypnea of the
level of 3,6 mmol/l. What agent newborn
could have caused the disease
in the child?
A. Enterovirus 47. During last several weeks
B. Neisseria meningitidis an 11-yearold girl has been
C. Mycobacterium tuberculosis complaining of dyspnea and
D. Staphylococcus aureus edema of shins and feet after
E. Streptococcus pneumoniae physical exercise. After a long
rest or sleep through the night
her edemas diminish headache in the frontoparietal
significantly. On clinical area, vertigo, palpitations. She
examination there are enlarged has been suffering from
liver and rasping systolic hypertension for 3 years.
murmur over the cardiac area. Significant increase of BP
Blood and urine analyses occurs 2-3 times per month and
arewithout changes. What is lasts for 3-8 hours. The left
the most likely cause of the ventricle is enlarged, heart
child’s edema? sounds are clear, heart rate -
A. Angioneurotic edema 105/min., BP- 225/115 mm Hg.
B. Heart failure ECG: signs of left ventricular
C. Acute pyelonephritis hypertrophy. What drug would
D. Hepatocirrhosis be the most effective for
E. Nephrotic syndrome termination of cerebral crisis
attack?
A. Hydrochlorothiazide
48. A 65-year-old man was (Hypothiazide)
diagnosed with B12-deficient B. Labetalol
anemia and the treatment was C. Captopril
prescribed. A week later D. Losartan
control blood test was E. Clonidine (Clophelin)
performed.What would be the
early indicator of the therapy
effectiveness? 50. A 10-year-old boy with
A. Normoblastic hematopoiesis symptoms of arthritis and
B. Increased hemoglobin level myocarditis was delivered into
C. Megaloblastic hematopoiesis a hospital. Based on clinical
D. Increased number of examination the preliminary
reticulocytes diagnosis of juvenile
E. Increased erythrocyte rheumatoid arthritis was made.
number What symptom is the most
contributive for the diagnostics
of this disease?
49. A 42-year-old woman A. Affection of the large joints
complains of severe pulsing
B. Regional hyperemia of the spines, and sacrococcygeal joint
joints can be palpated.Where in the
C. Reduced mobility of the lesser pelvis is the fetal head
joints in the morning situated?
D. Enlarged heart A. Above pelvic inlet
E. Increased heart rate B. In the wide plane of lesser
pelvis
C. In the narrow plane of lesser
51. A 23-year-old man had pelvis
taken 1 g of aspirin to treat D. In the area of brim
acute respiratory infection. E. In the area of pelvic outlet
After that he developed an
asthmatic fit with labored
expiration that was arrested by 53. A 3-year-old girl is being
introduction of aminophylline. treated at a resuscitation unit
The patient has no medical with diagnosis ”acute kidney
history of allergies. The patient failure, oligoanuric stage”. ECG:
has undergone two surgeries high T wave, extended QRS
for nasal polyposis in the past. complex, displacement of S-T
What is the most likely interval downwards below the
diagnosis? isoline. What electrolyte
A. Exercise-induced asthma imbalance is it?
B. Atopic bronchial asthma A. Hyperkalemia
C. Infectious allergic bronchial B. Hypokalemia
asthma C. Hypocalcemia
D. Aspirin-induced asthma D. Hypercalcemia
E. Symptomatic bronchospasm E. Hyperphosphatemia

52. Vaginal examination reveals 54. Examination of a 43-year-


the head of the fetus, which old man objectively revealed
fills the posterior surface of pallor of skin and mucous
symphysis pubis and hollow of membranes, loss of tongue
the sacrum. The lower edge of papillae, transverse striation of
symphysis pubis, ischiadic fingernails, cracks in the mouth
corners, tachycardia. Blood test day the patient persistently
results:Hb- 90 g/l, anisocytosis, presents with temperature up
poikilocytosis. The most likely to 40oC, heart rate is 110/min.,
causative factor of this respiration rate is 23/min., BP is
condition is the inadequate 100/60 mm Hg. Blood test:
intake of: leukocytes - 16 · 109/l, band
A. Magnesium neutrophils - 16%. What phase
B. Copper of clinical course is it?
C. Zinc A. Rehabilitation
D. Iron B. Anabolic
E. Selene C. Catabolic
D. Functional
E. Tension
55. A multigravida on the 38th
week of her pregnancy
complains of increased BP up to 57. A 35-year-old patient
140/90mmHg, edema of the complains of heartburn,
shins for 2 weeks. In the last gasseous and sour eructation,
month she gained 3,5 kg of burning constricting pain
weight. Urine analysis: protein - behind the sternum and along
0,033 g/l. Make the diagnosis: the esophagus, developing
A. Moderate preeclampsia when bowing the torso to the
B. Mild preeclampsia front. No previous examination;
C. Pregnancy hypertension the patient takes almagel at his
D. Severe preeclampsia own discretion, after which he
E. Pregnancy edema notes improvement of his
general state. Make the
provisional diagnosis:
56. A 43-year-old man has A. Gastroesophageal reflux
undergone a surgery for disease
osteomyelitis of the left thigh. B. Functional dyspepsia
On the 6th day the patient’s C. Cardiospasm
condition was complicated with D. Ulcer disease of the stomach
sepsis. Despite complex E. Ulcer disease of the
therapy of sepsis on the 9th duodenum
tongue is thickened. Heart
sounds are muffled. HR-
58. A 15-year-old girl complains
56/min, BP- 140/100 mm Hg.
of dizziness and sensation of
The patient has been suffering
lack of air that she develops in
from amenorrhea for 5 months,
emotionally straining
has constipations. TSH- 28
situations. Relief occurs after
mcIU/l (norm is 0,32-5).
she takes corvalol. Objectively:
Craniogram shows no
hyperhidrosis and marble-like
pathology. What is the etiology
pattern of the skin of her palms
of obesity?
and feet. Clinical and
A. Hypothalamic-pituitary
instrumental examination
B. Hypo-ovarian
revealed no organic alterations
C. Hypothyroid
of the central nervous,
D. Alimentary and constitutive
cardiovascular, and respiratory
E. Hypercorticoid
systems. What provisional
diagnosis can be made?
A. Stenosing laryngotracheitis
60. A multigravida at 39 weeks
B. Obstructive bronchitis
of gestation presenting with
C. Bronchial asthma
regular labour activity for 8
D. Somatoform autonomic
hours has been delivered to a
dysfunction
hospital; the waters broke an
E. Acute epiglottitis
hour ago. She complains of
headache, seeing spots. BP is
180/100 mm Hg. Urine test
59. A 35-year-old woman has
results: protein - 3,3 g/l, hyaline
gained 20 kg weight within a
cylinders. Fetal heart rate is
year with the normal diet. She
140/min, rhythmical. Vaginal
complains of chills, sleepiness,
examination reveals complete
shortness of breath. The
cervical dilatation, the fetal
patient’s mother and sister are
head is on the pelvic floor,
corpulent. Objectively: height -
sagittal suture is in line with
160 cm, weight - 92 kg, BMI-
obstetric conjugate, the
35,9. Obesity is uniform, there
occipital fontanel is under the
are no striae. The face is
amimic. The skin is dry. The
pubis.What is the optimal area, Which of the following
tactics of labour management? additional skin examination
A. Cavity forceps findings would be most
B. Outlet forceps conceming for malignancy?
C. Cesarean section
D. Vacuum extraction of the A. Central keratin plug
fetus B. Golf ball-size subcutanecus
E. Conservative labour mass
management C. Multicolored lesion
D. Perphersi scaling
E. "Stuck-on" appearance
61. A 32-year-old woman
comes to the office due to a
62. A 56-year-old woman
'mole." The lesion is located on
comes to the office with skin
her back, and she believes it
lesions on her right arm that
has been present for years but
she noticed 3 weeks ago. She
does not recall when it first
underwent a right mastectomy
appeared. The patient thinks it
for advanced breast cancer 7
might be enlarging and is
years ago and was treated with
concerned due to a family
radiation and chemotherapy.
history of skin cancer. The
Since her treatment, the
lesion is not painful and does
patient has had swelling of the
not itch or bleed. She has fair
right arm. Her mother was
skin and a history of severe
recently diagnosed with
sunburns in childhood. Medical
shingles. Family history is
history is unremarkable, and
significant for breast cancer in
she does not use tobacco,
her aunt. BRCA1 and BRCA2
alcohol, or ilicit drugs. Vital
testing was negative. Physical
signs are nomal. Skin
examination shows nonpitting
examination shows a few
edems of the right upper
scattered benign-appearing
extremity. There are 3 firm
nevi and a single prominent
purple papules on the inner
lesion at her right scapular
surface of the right arm. Which
of the following is the best next D. Melanoma
step in management of this E. Squamous cell carcinoma
patient?
A. Antifungal therapy 64. A 6-month-old boy is
B. Lesion biopsy brought to the office for
C Low-potency topical steroid evaluation of a progressive
cream facial rash that started 3 weeks
D. Oral valacyclovir ago near his eyebrows and
E. Reassurance and observation spread to areas of the face and
scalp. The rash does not appear
to be painful or pruritic. The
63. A 65-year-old man comes to
the office 4 years after a burn mother recently started using a
mild. unscented shampoo that
injury to his left leg that
slightly Improved the scalp
required extensive local care
and skin grafting, resulting in rash. In addition,
erythematous. Scaly plaques
significant residual scar tissue.
are on the eyebrows, nasolebial
During treatment of the burn,
the patient also developed a folds, and behind the ears.
Which of the following is the
chronic draining wound that
most likely diagnosis?
never completely closed. For
the last 6 weeks, he has had an A. Atopic dermatitis
enlarging nodule at the lesion B. Contact dermatitis
site associated with persistent C. Psoriasis
pain and increased drainage. D. Seborrheic dermatitis
Treatment with topical therapy E Tinea capitis
has not helped. On biopsy of
the nodule, which of the
65. A 6-day-old infant is
following is most likely to be brought to the emergency
Identified?
department by his mother for
A. Basal cell carcinoma lethargy and poor feeding. The
B. Cutaneous T-cell lymphoma mother's medical history is
C. Kaposi sarcoma unremarkable. The pregnancy
and delivery were uneventful, the vaginal walls, but no
but she did not have adequate erethima or edema is present
prenatal checkups and care. on the vaginal walls or the
Physical examination of the vulva. There is no cervical or
infant shows hypotonia, poor adnexal tenderness. A saline
reflexes. and bulging wet mount examination reveals
fontanelles. Temperature is numerous epithelial cells
39.4° C (103° F). Initial coated with bacteria. No white
laboratory studies show a white blood cells or motile organisms
blood cell count of 16.000/ul are seen. Which of the
with 20% bands. What is the following is the best
most likely cause of this infant's management option for this
condition? patient?
A. Group B Streptococcus A. Boric acid vaginal
B. Haemophilus influenzae suppository
C. Herpes simplex virus B. Delay treatment until 2nd
D. Listeria monocytogenes trimester
E. Neisseria meningitidis C. Oral azithromycin
F. Toxoplasma gondi D. Oral Metronidazole
E. Povidone-iodine vaginal
66. A 25 year old women, douche
F. Tropical fluconazole
gravida 1 para 0, at 10 weeks
gestation comes to the office
due to a malodorous vaginal 67. A 25-year-cld woman
discharge. He is sexually active, presents to your office
has no chronic medical complaining of a seven-week
conditions, and has no history of amenorrhea. She also
medication allergies. Vital signs states that she has had nausea
are normal. BMI is 26 kg/m2. and vomiting for five weeks,
Fetal heart tones are 160/min She is sexually active. Her
speculum examinations reveals medical and obstetrical
a thin, grey discharge that cotes histories are unremarkable
Serum hCG level is elevated. condition is most likely
Which of the following is the associated with the patients
most important direct role of finding?
hCG in pregnancy?
A. Adenocarcinoma
A. Inhbition of uterine B. Fibroadenoma
contractions C. lymphoma
B. Induction of prolactin D. papilloma
production by the pituitary E. squamous cell carcinoma
C. Promotion and maintenance
of implantation
69. An 18 year old women
D. Maintenance of the corpus
comes to the office due to
luteum lower abdominal pain that
E. Induction of early embryonic
radiates to her lower back and
division and differentiation
thighs during menses. The
patient had no pain while on
68. A 53 year old woman comes oral contraceptives but stopped
to the office with strange, itchy taking them 5 months ago due
rash on her left breast, which to unscheduled breakthrough
has been present for last bleeding. Menstrual periods
month. The patient applied now occur every 30 days with 5
over the counter corticosteroid days of bleeding the pain and
ointment on to this rash with bleeding are worse on the 2nd
no relief of symptoms. She day, requiring the patient to
takes no medication. The change her pad every 4 hours.
patients last menstrual periods She also has nausea and fatigue
was 2years ago. She has a during the first 2 days of
history of hypertension that menses. The patient is sexually
improved with weight loss and active with a male partner and
exercise. Physical examination uses condoms in constantly.
shows an eczematous plaque She has no intermenstrual
on the left nipple and areola bleeding or pain with
which of the following intercourse. Vital signs are
normal. BMI is 19.5 kg/m2. takes no daily medications and
Pelvic examination reveals a does not use tobacco, alcohol
small, mobile uterus and no or illicit drugs. Blood pressure is
palpable adnexal masses. 124/70 mm hg and pulse is
Which of the following is the 88/min. BMI is 29 kg/m2. There
most likely diagnosis in this is coarse hair on the upper lip
patient? and bitemporal hair thinning.
A. Adenomyosis Examination reveals a large
pelvic mass extending
B. Endometriosis
C. Intermittent ovarien torsion throughout the left lower
quadrant, but no tenderness or
D. Pelvic congestion syndrome
ascites. The cervix appears
E. Primary dysmenorrhea
F. Uterine leiomyoma multiparous with no active
bleeding. Urine pregnancy test
is negative. Which of the
70. A 38 year old women, following the most likely
gravida 3 para 3, Comes to the diagnosis in this patient?
office for evaluation of
abnormal uterine bleeding. The A. Choriocarcinoma
B. Ovarian thecoma
patient had regular monthly
C. Polycystic ovary syndrome
menses until 6 months ago,
when she began to have inter D. Sertoli leidig cell tumor
E. Struma ovarii teratoma
mensural spotting and has now
F, Uterine Leiomyoma
been amenorrheic for 3
months. She has had increased
abdominal bloating and 71. A 26-year-old woman came
cramping during the last few to a gynaecologist for are gular
months but no heart flushes or check-up. She has no
weight changes. The patient complaints. Pervaginum: the
has no chronic medical uterus lies in ante flexion, not
conditions and underwent a enlarged, dense, mobile,
bilateral tubal ligation after her painless. On the left from the
last delivery 2 years ago. She uterus in the area of uterine
appendages there is a mobile joint D. Closed dislocation of
painless out growth that can be the left hip E. Closed cervical
moved independently from the fracture of the left hip
uterus. On the right the
appendages cannot be
detected. What additional 73. A 7-year-old child complains
investigation would be of cramping pain occuring after
mental exertion, cold drinks
informative for diagnosis
and eating ice-cream.
clarification?
Instrumental examination
A. Ultrasound of the lesser allowed to diagnose biliary
pelvis B. Metro dyskinesia of hypertensive
salpingography C. type. What group of drugs
Examination for urogenital should be prescribed for
infection treatment?
D. Colposcopy A. Sedatives and cholekinetics
B. Choleretics and cholekinetics
E. Colonoscopy
C. Antispasmodics and
choleretics
D. Antioxydants
72. When her car collided with
E. Antibiotics
a tree, a 37- year-old woman
felts harp pain in her left hip
joint. She was unable to get out 74. 40-50 minutes after the
of the car. Her position is completion of repair works
forced, the hip is pressed to the conducted in a closed garage,
abdomen, fixed, and rotated with car engine running, the
inwards; the limb is flexed in repair workers developed
the knee, any attempt to severe head ache in the
change the position results in temporal area, nausea, tinnitus,
sharp pain. Make the diagnosis: vertigo, etc. These symptoms
A. Arthritis of the left hip joint are characteristic of acute
B. Contusion of the left hip joint poisoning with:
C. Hemarthrosis of the left hip
A. Aldehydes areas with irradiation to the
B. Carbon monoxide back, nausea, and vomiting
C. Organochlorides without relief. On examination
D. Hydrogen sulfide a surgeon observes stomach
E. Fluoride distension and meteorism.
There are positive Mondor’s
Mayo-Robson’s, and Cullen’s
75. A 37-year-old woman symptoms. What is the most
complains of headaches, likely diagnosis?
nausea, vomiting, spasms. The A. Aortic dissecting aneurysm
onset of the disease occurred B. Acute cholecystitis
the day before due to her C. Acute intestinal obstruction
overexposure to cold. D. Acute pancreatitis
Objectively: fever up to 40oC; E. Splenic infarction
somnolence; rigid neck;
Kernig’s symptom is positive on
the both sides; general 77. A resuscitation unit
hyperesthesia. Blood test: received a 46-year old woman,
leucocytosis, increased ESR. who has been suffering from
Cerebrospinal fluid is turbid, diabetes mellitus type 1 for
yellow-tinted. What changes of approximately 30 years.
the cerebrospinal fluid are Objectively: the skin is pale,
most likely? heart sounds are weakened, BP
A. Neutrophilic pleocytosis is 170/100 mm Hg, lower limbs
B. Lymphocytic pleocytosis are markedly swollen. Blood
C. Blood in the cerebrospinal creatinine - 1125 mcmol/l, urea
fluid - 49,6 mmol/l, potassium - 6.3
D. Xanthochromia in the mmol/l, glucose - 7,6 mmol/l,
cerebrospinal fluid glomerular filtration rate - 5
E. Albuminocytological ml/min. What treatment is
dissociation indicated for the patient in the
first place?
76. A 28-year-old woman A. Kidney transplantation
complains of girdle pain in her B. Hemodialysis
epigastric and left subcostal C. Hemofiltration
D. Enterosorption expert committees, Ministry
E. Conservative detoxification of Defencemedical
committees, Ministry of
therapy
Domestic Affairs medical
committees, forensic
medicine agency, etc. Specify
78. A 43-year-old man, who has what service deals with
been abusing alcohol and sociomedical assessment of
suffering from pulmonary temporary disability:
tuberculosis, in the course of A.Medicalandpreventive
two weeks gradually developed treatmentfacilities
B.Hygiene and preventive
general weakness, headache, treatment facilities
diplopia, vomiting. Objectively: C.Sociomedical expert
ptosis on the left, anisocoria committees
S>D, exotropia of the left eye, D. Ministry of Defence medical
neck stiffness; Kernig’s and committees
Brudzinski’s signs are positive. E.Ministry of Domestic Affairs
In cerebrospinal fluid: medical committees
lymphocytic pleocytosis, low
glucose, precipitation of 80. A woman with blood group
cerebrospinal fluid resulted in B (III)Rh(+) gave birth to a full-
production of fibrin film. What term healthy boy. Examination
is the most likely diagnosis? on the 3rd day of the infant’s
A. Acute myelitis
life shows him to have
B. Subarachnoid hemorrhage
icterictint to his skin. The child
C. Brainstem encephalitis
D. Tuberculous meningitis has no problems with suckling,
E. Basal arachnoiditis sleep is non disturbed. The
abdomen is soft, the liver
protrudes by 2cm from under
79. Establishments the costal margin. Complete
participating in medical blood count: hemoglobin-
examinations include:
200g/L, erythrocytes- 5.5 ·
medical and preventive
treatment facilities, hygiene 1012/L, total bilirubin-
and preventive treatment 62mcmol/L, indirect bilirubin-
facilities, social medical
52 mcmol/L. What condition C. Right-sided renal colic
can be suspected? D. Perforative ulcer
E. Acute appendicitis
A. Hemolytic disease of the
new born due to Rh
incompatibility B. 82. Infant is 6.5 months now
Congenital hepatitis and is given natural feeding
C. Physiologic jaundice since birth. Body mass was 3.5
D. Biliary atresia kg, with length 52 cm at birth.
E. Hemolytic disease of the new How many times per day the
born due to ABO supplement (up feeding) should
incompatibility be given?
A. 4
B. 0
81. A 43 y.o. woman complains
of severe pain in the right C. 2
D. 3
abdominal side irradiating in
E. 1
the right supraclavicular area,
fever, dryness and bitterness in
the mouth. There were 83. During a surgery on a 30-
multiple vomitings without year-old patient a dark ileoileal
relief. Patient relates the onset conglomerate was discovered,
of pain to the taking of fat and the intussusceptum intestine
fried food. Physical was considered to be
examination: the patient lies on unavaiable. The
the right side, pale, dry tongue, intussuscipiems intestine was
tachycardia. Right side of dilated to 7-8 cm, swollen, full
abdomen is painful during the of intestinal contents and
palpation and somewhat tense gases. What pathology led to
in right hypochondrium. What the surgery?
is the most likely diagnosis?
A. Spastic obstruction
A. Acute cholecystitis B. Paralytic obstruction
B. Acute bowel obstruction
C. Invagination (combined) D. Nephrolithiasis
obstruction E. Chronic pyelonephritis
D. Strangulation obstruction
E. Obturation obstruction 85. A 12-year-old boy has a 6-
year history of insulin-
dependent diabetes. The
84. A 54-year-old male patient
disease is labile. Since recently
complains of aching pain in the
lumbar region that is getting there have been periodical rises
in blood pressure.
worse after standing in an
upright position, physical Microalbuminuria test gave
positive results. The patient's
exercise, super cooling. The
condition corresponds with the
patient also reports of
experiencing weakness in the following stage of diabetic
nephropathy:
afternoon. Pain in the lumbar
region, said about 10 years old. A. Stage II-histological changes
Objectively: pale skin, tº- 37, in the kidneys
2°C, AP- 180/100 mm Hg, minor B. V stage- chronic renal failure
costovertebral angle C. Stage I-renal hypertrophy
tenderness (Pasternatsky and hyperfunction
symptom). In blood: RBCS – 3, D. Stage III-early-stage
5· 1012/1, WBCS – 6, 5 · 109/l, nephropathy
ESR – 22 mm/h. In urine: the E. Stage IV-advanced clinical
relative density 1010, nephropathy
leukocytes - 12-15 in the field
of vision, erythrocytes 2-3 in
86. A 38-year-old woman
the field of vision. Urine experiences episodic increases
bacterial count
in arterial pressure up to
100000 in 1 ml. What is the 240/120 mm Hg, which is
most likely diagnosis? accompanied by nausea,
vomiting, tachycardia,
A. Amyloidosis
B. Polycystic renal disease increased sweating,
hyperglycemia. The attack is
C. Chronic glomerulonephritis
usually followed by the observed. Pulse - 10/min. blood
excessive urination. Renal pressure 210/130 mmHg,
sonography reveals an respiration rate - 38/min.
additional formation adjacent Pulmonary percussion sound is
to the upper pole of the right clear, with slight dullness in the
kidney and possibly belonging lower segments; throughout
to the adrenal gland. What the lungs single dry crackles can
laboratory test will allow to be heard that become bubbling
clarify the diagnosis? and non-resonant in the lower
A. Blood test for renin level segments. What is the most
likely complication in this
B. Estimation of glomerular
patient?
filtration rate by measuring
endogenous creatinine A. Paroxysmal tachycardia
clearance B. Bronchial asthma attack
C. Blood test for thyroxine and C. Acute right ventricular failure
thyrotrophic hormone D. Acute left ventricular Failure
D. Determination of urinary E. Pulmonary embolism
excretion of catecholamines
and vanillylmandelic acid
88. A 45 year old man was
E. Blood test for insulin and C-
brought by an ambulance into
peptide the emergency hospital. He
complains of sudden pain in the
87. At night a 63-year-old humbar area, frequent painful
woman suddenly developed an urination and vomiting.
asphyxia attack. She has a 15 Examination detects pain in the
year long history of essential lumbar area, costovertebral
hypertension and has a angle tenderness, pain on
myocardial infarction 2 year palpution of kidneys and along
ago. Objectively her position in the ureter on the right. Urine
bed is orthopneic, the skin is test: proteins, fresh
pale, the patient is covered erythrocytes, leukocytes. Make
with cold sweat, acrocyanosis is the provisional diagnosis:
A. Urolithiasis, renal colic E. Ultra sound
B. Acute glomerulonephritis 90. A patient complains of
C. Polycystic kidney disease
suppuration from the ear and
D. Acute renal failure
impaired hearing of the left ear,
E. Acute pyelonephritis which have been observed for
the past 6 years. The patient
89. After excessive had periodical headaches,
consumption of fatty food a 60- general indisposition, fever.
year-old woman suddenly Objectively: otoscopy of the
developed pain in herright external auditory meatus
subcostal area, nausea, revealed ucopurulent odorless
bilevomiting, sharp bitter taste substance. The eardrum is of
in her mouth. In 2 days she normal color, with central
developed jaundice,her urine perforation. What is the most
darkened. Objectively: sclera likely diagnosis?
and skin are icteric, the
A. Chronic mesotympanitis
abdomen is distended, the liver
B. Otosclerosis
is enlarged by 3 cm,soft and C. Acute otitis media
painful on palpation, Ortner’s,
D. Chronic sensorineural
Murphy’s, Kehr’s, Zakharyin’s,
hearing loss
Mayo-Robson’s signs are E. Chronic epitympanitis
positive. What diagnostic
technique should be used in
the first place to confirm the 91. A primigravida at the term
diagnosis? of 20 weeks complains of pain
in her lower abdomen,
A. Laparoscopy
smearing blood-streaked
B. Esophago duodenoscopy discharge from the genital
tracts. Uterine tone is
C. Abdominal X-ray
increased, fetus is mobile. On
D. Radionuclide scanning of the vaginal examination: the uterus
liver and gallbladder is enlarged according to the
term, uterine cervix is
shortened to 0,5 cm, external
cervical orifice is open by 2 cm. 93. A 54-year-old patient has
What is the most likely
been suffering from diabetes
diagnosis?
mellitus for 5 years, with diet
A. Risk of late abortion without being his only treatment.
hemorrhage Within the last half a year he
B.The process of late abortion lost 7 kg of body weight,
C. Risk of late abortion with complains of thirst, vertigo
hemorrhage when raising from bed,
D. Incomplete late abortion decrease of erectile function,
E. Attempted late abortion frequent stool, specially at
night. Objectively: malnutrition,
dry skin. BP in lying position is
92. A parturient woman is 23
160/90 mm Hg; BP in standing
years old. Internal obstetric
examination shows the uterine position is 170/85 mm Hg. No
edemas. Fasting plasma glucose
cervix to be completely open.
level is 12 mmol/l. Glycated
Fetal bladder is absent.
Cephalic presentation is haemoglobin accounts for 11%.
Albumin excreted with urine is
observed in the plane of the
20 mg per day. The most likely
small pelvic outlet. Sagittal
suture is at the longitudinal diagnosis is:
section of the small pelvic A. Diabetes mellitus type 1
outlet, small fontanel is with encephalomyelopathy
situated closer to the uterus. B. Diabetes mellitus type 1
What cephalic position will the with ketoacidosis
newborn have during birth in C. Diabetes mellitus type 2
this case? with visceral neuropathy
A. Minor oblique lie D. Diabetes mellitus type 2
with nephropathy
B. Longitudinal lie
E. Diabetes mellitus type 2
C. Transverse lie
D. Medium oblique lie with polyneuropathy
E. Major oblique lie
94. A 5 y.o. child with stigmas E Corglicone
of dysembryogenesis (small
chin, thick lips, opened mouth,
96. A 34 year old man
hyperthelorismus) has systolic
complains of pale edema of the
murmur in the second face, feet, shins and lumbar
intercostal to the right of the
area, elevated BP upto 160/100
sternum. The murmur passes to
mmHg, und general weakness.
the neck and along the sternum He has a clinical history of
left edge. The pulse on the left
nonspecific ulcerative colitis.
brachial artery is weakened. BP Objectively: pulse - 84/min,
on the right arm is 110/60 mm
rhythmic, blood pressure –
Hg, on the left - 100/60 mm Hg.
165/100 mmHg; edemas all
ECG results: hypertrophy of the over the body; the skin is pale
right ventricle. What defect is
and dry, with low turgor. The
the most probable?
kidneys cannot be palpated, on
A Aortic stenosis an attempt to palpate them
B Defect of interventricular they are painless. Blood test:
septum erythrocytes - 3.0 x 1012/L, Hb -
C Defect of interatrial septum 100 g/L, erythrocyte
D Coarctation of the aorta sedimentation rate - 50
E Open aortic duct mm/hour. Urinalysis: proteins -
3.5 g/L, erythrocytes - 7-10 in
the vision field, leukocytes - 5-6
95. A 14 year old child suffers
from vegeto vascular dystonia in the vision field. Daily
proteinuria – 6 grams. What
of pubertal period. He has got
sympatho adrenal attack . analysis should be conducted
additionally to verify the
What medicine should be used
diagnosis?
for attack reduction?
A. Radio isotopic examination
A Obsidan
of kidneys
B No-shpa
C Amysyl B. Gingival biopsy for the
diagnosis of amyloid disease
D Aminophylline
C. Renal ultrasound D. Yes, he developed a
D. Survey and excretory moderate form of acute
urography radiation syndrome
E. Urinalysis for Bence-Jones E. No, he did not
protein
98. Indicators of work of
97. A 20 year old man, a impatient departments in the
city hospitals for the past year
calibrator of dosimetry
equipment, committed a gross were analysed. After that the
meeting was held at the central
violation of safety regulations,
city hospital and based on the
when he put two ampoules
with cobal-60, each with the results of the analysis, a
decision was made to decrease
radioactivity of 7 MCI, in the
the number of beds in the
pockets of his trousers and
jacket. He has been keeping the impatient departments and
instead open day care units
ampoules in his pocket for 8
with partial hospitalization in
hours. The tissue at the
distance of 0.5 cm from the the city poly clinics. What is the
main goal of this decision?
source received the dose of 30
Gy (3000 R), while the tissues at A. Decreased mortality during
the distance of 20 Cm – 2 R. Did impatient treatment
this man develop radiation B. Optimization of the average
sickness? bed occupancy rate per year
C. Decrease of the length of
A. Yes, he developed a severe
form of acute radiation impatient stay
D. Rational use of the bed fund
syndrome
E. Intensification of the bed
B Yes, he developed chronic
radiation syndrome fund usage
C.Yes, he developed a mild
form of acute radiation 99. A boy is 8 year old. His
syndrome physical development is
compliant with his age. The
child has had cardiac murmur 0.5 cm, the externmal orifice
since birth. Objectively: skin was open by 2 cm. The
and visible mucous membranes discharges were bloody and
are of normal colour. AP- smeary. What is the most likely
100/70 mm Hg. Auscultation diagnosis?
revealed systolo-diastolic
A. Missed miscarriage
murmur and diastolic shock
B. Incomplete abortion
above the pulmonary artery. C. Abortion in progress
ECG shows overload of the left
D. Risk of abortion
heart. Roentgenoscopy shows E. Incipient abortion
coarsening of the lung pattern,
heart shadow of normal form.
What is the most likely 101. A 32 weeks premature,
diagnosis? healthy infant weighing less
than 1 kg is having jaundice
A. Patent ductus arteriosus with serum bilirubin level of 5
B. Pulmonary artery stenosis
mg%. Ideal management in
C. Atrioseptal defect
such a case would be:
D. Fallot's tetrad
E. Aorta contraction a. Observation
b. Phototherapy
c. Medical management
100. A primagravida in her 20th
d. Exchange transfusion
week of gestation complains
about pain in her lower
abdomen, blood smears from 102. A 30-year-old lady
the genital tracts. The uterus delivered a small for age child
has an increased tonus, the at 38 weeks of pregnancy On
patient feels the fetus examination, the new born was
movements. Bimanual having rhinitis, abdominal
examination revealed that the distension, wrinkled skin and
uterus size corresponded the maculopapular rash. There is
term of gestation, the uterine past history of abortions and
cervix was contracted down to
stillbirths. Presentation is primigravida with placenta
suggestive of: praevia and active bleeding:
A. Congenital rubella a. Labor induction
B. Congenital toxoplasmosis b. Cesarean section
C. Congenital syphilis c. Vaginal delivery
D. Neonatal pemphigus d. Expectant treatment

103. Hereditary fructose 106. Which of the following is


intolerance is due to the known as pacemaker of uterine
deficiency of: contractions?
A. Phosphofructokinase a. Fundus
B. Xylitol dehydrogenase b. Anterior uterine wall
C. Aldolase B С. Posterior uterine wall
D. Fructokinase d. Tubal ostia

104. A 5-year-boy presented 107. Which of the following


with massive hematemesis. He types of placenta complicates
is NOT febrile. Per abdomen third stage of labor and is
examination revealed massive associated with a past history
splenomegaly without of cesarean section:
hepatomegaly. Most probable a. Placenta succenturita
diagnosis would be:
b. Placenta extracholis
a. Non-cirrhotic portal fibrosis c. Placenta membranecae
b. Extrahepatic portal venous d. Placenta accrete
obstruction
c. Budd-Chiari syndrome 108. Golf club appearance of
d. Hepatic carcinoma fallopian tube on
hysterosalpingography suggest
the diagnosis of:
105. Ideal management of a 37
weeks pregnant elderly a. ТВ
b. Gonorrhea years the boy has been
c. Chlamydial infection registered for regular
check-ups due to his
d. None of the above diagnosis of third degree
persistent bronchial
asthma. To provide
109. A 46-year-old lady has
emergency aid for this
abnormal uterine bleeding with child, first he needs to be
simple hyperplasia of given:
endometrium. The treatment A. Adrenaline
B. Dexamethasone
of choice would be: C. Salbutamol or short-
acting β2-agonists
a. Total hysterectomy D. Euphylline
(Aminophylline)
b. Progesterone E. Claritin (Loratadine)
c. Estrogen+ progestogen
d. Estrogen 112. A 6-month-old child on
breastfeeding is
110. 18-year-old girl presented hospitalized in the inpatient
department. After the child
with primary amenorrhea,
recovers, the doctor
normal breast development, recommends the mother to
absent pubic and axillary hair. start introducing solid food
Condition is: to the child’s diet. What
products should be
a. Turner syndrome introduced to the child’s
b. Mullerian agenesis diet first?
c. Premature ovarian failure A. Semolina porridge
B. Fermented dairy products
d. Testicular feminizing C. Grated apple
D. Vegetable puree
syndrome E. Buckwheat porridge

113. A newborn has Apgar


111. A 10-year-old boy was score of 9. When should
brought into the hospital this infant be put to the
with complaints of breast?
expiratory dyspnea, A. In the delivery room
B. After 12 hours
respirations are 30/min. He C. After 2 hours
explains his state by a D. On the 2nd day
change in the weather E. On the 3rd day
conditions. For the last 4
114. A 28-year-old man, a A. Recombinant influenza
teacher, after emotional stress vaccine(RIV)
developed painful muscle B. Inactivated influenza
spasms in his right hand that vaccine(IIV)
occur during writing; now he C. Type of influenza vaccine is
has to hold the pen between not important
the second and third fingers. D. Vaccination is
He has no problems with typing contraindicated due to severe
or writing on the black board; heart failure E.
no other motor disturbance Vaccination is contra indicated
sorne urological pathologies due to elderly age of the
are detected. What is the most patient
likely diagnosis? 116. During administration of
A. Neuropathy of the right planned DPT vaccination the
ulnar nerve child suddenly developed acute
B. Cortical agraphia anxiety, signs of pain response,
C. Parkinsonism dyspnea, grunting respirations,
D. Writer’s cramp cut is marmorata, cold sweat.
E. Neuropathy of the right Objectively the child’s
radial nerve consciousness is disturbed,
115. A 69-year-old woman was heart rate is 150/min., blood
pressure is 60/40mmHg, hearts
diagnosed with the following:
pounds are muffled. The child
ischemic heart disease; stable
exertional angina pectoris, FC was diagnosed with
anaphylactic shock. What drug
III; heart failure II A with
retained left ventricular should bead ministered first?
rejection fraction, functional A. Euphylline (Aminophylline)
class III (NYHA). What vaccine B. Lasix(Furosemide)
should be chosen for influenza C. Suprastin (Chloropyramine)
prevention and to avoid D. Epinephrine
destabilization of the patient’s E. Analgin (Metamizole)
condition?
117. A 60-year-old woman joints D. Enlarged
complains of unbearable pain heart E.
in her right subcostal area. She Increased heartrate
has a history of acute
119. A 36-year-old man has
pancreatitis. Temperature is been complaining of marked
38.2oC. Objectively her sclera
weakness, low appetite, and
are icteric. There are no signs of
nausea for the last year, year
peritoneal irritation. Ortner’s and a half. With in the last year
and Gubergrits’ symptoms are
he has lost 10kg of body mass.
positive. Urine diastase is The skin is darkened, especially
320U/L.What is the most likely
on his face, neck, and arms.
diagnosis?
Skin fold sand nipples are hyper
A. Acute cholangitis pigmented, the re are pigment
B. Exacerbation of chronic spots on the patient’s inner
pancreatitis thighs. Pulse is 60/min., blood
C. Chronic cholecystitis pressure is 80/50mmHg.What
D. Acute cholecystitis is the provisional diagnosis?
E. Pancreatic cancer A. Chronic gastritis
118. A 10-year-old boy with B. Diabetes mellitus
symptoms of arthritis and C. Adrenal insufficiency
myocarditis was delivered into D. Hemochromatosis
a hospital. Based on clinical E. Cholestatichepatitis
examination the preliminary 120. Due to introduction of a
diagnosis of juvenile
new treatment method,
rheumatoid arthritis was made. average duration of the rapying
What symptom is the most
the experimental group was
contributive for the diagnostics
12.3±0.2 days compared
of this disease? to15.4±0.4 days in the control
A. Reduced mobility of the group that was treated by the
joints in the morning old method. What calculations
B. Regional hyperemia of the should be made to estimate the
joints C. Affection of the large
statistical significance of the T wave and deforms it,
difference in the results? moderate lengthening of PQ
interval, QRS complex is
A. Sign test(Z-test)
without alterations. What
B. T-test (Student’st- pathology does this child have?
distribution) C. Matching A. WPW syndrome
factor(chi-squaredtest) D. B. Atrial hypertrophy
WilcoxonT-test C. Ventricular hypertrophy
E. Kolmogorov-Smirnovtest D. Paroxysmal atrial
tachycardia
E. Extra systole
121. Survey radiograph of a 52-
123. A 3-month-old child
year-old worker of an
presents with saffron-yellow
agglomeration plant (28-year
coloring of the skin, sclera, and
long record of service, the
mucous membranes. The
concentration of metal dust is
abdomen is enlarged,
22-37 mg/m3) shows mildly
hepatomegaly and
pronounced interstitial fibrosis
splenomegaly are observed. In
with diffused contrasting well-
blood there is conjugated
defined small nodular shadows.
bilirubin-induced
The patient has no complaints.
hyperbilirubinemia. On
Pulmonary function is not
intravenous
compromised. What is the
cholangiocholecystography:
provisional diagnosis?
opacified bile is discharged into
A. Anthraco-silicatosis
the intestine. Transaminase
B. Silicosis
activity is normal. What is the
C. Siderosis
most likely diagnosis?
D. Silicatosis
A. Physiologic jaundice
E. Anthracosis
B. Biliary atresia
C. Hemolytic disease of
newborn
122. ECG revealed the following
D. Crigler-Najjar syndrome
in a 10-year-old child: sharp
E. Congenital hepatitis
acceleration of the heart rate -
240/min., P wave overlaps with
124. A woman in her early- to B. Vertebrogenous radicular
mid-thirties has lost her syndrome of L5-S1 on the left
consciousness 3-5 minutes ago. C. Left-sided coxitis
On examination: the skin is D. Left sided Vertebrogenous
pale, no pulse over the carotid lumbar ischialgia
arteries, no spontaneous E. Spinal stroke
respiration, pupils are dilated;
the patient is nonresponsive,
126. A 35-year-old patient
presents with atony. The
developed an epileptic attack
patient’s condition can be
determined as: with tonoclonic spasms that
A. Syncope lasted for 3 minutes. After the
B. Natural death attack the patient fell asleep
C. Apparent death but in 5 minutes the second
D. Brain death attack occurred. The first step
E. Coma of emergency aid would be to:
A. Take blood from the vein for
125. A 43-year-old woman analysis
complains of pain in the lumbar B. Ensure patency of airways
area, which irradiates to her C. Introduce diazepam
left leg and aggravates on intravenously
movement, and sensation of D. Prescribe antiepileptic drugs
numbness in this leg. E. Administer chloral hydrate
Objectively palpation of her via enema
shin and thig his painful, there
are painful stretch symptoms of 127. Estimation of community
on the left and gastrocnemius health level involved analysis of
cramps. There is no sensory are port on diseases registered
loss or weakening of reflex among the population of
responses. Make the diagnosis: district under charge (reporting
A. End arteritis of the lower form12).What index is
extremities calculated based on this
report?
A. Prevalence from her breasts and no
B. Index of pathological menstruation for the last 5
affection C. Index of months. One physiologic child
morbidity with temporary birth was 4 years ago. There are
disability D. no mal developments of
Index of hospitalized morbidity mammary glands. Bi- manual
E. Index of basic non-epidemic examination revealed
morbidity diminished uterus and normal
sized ovaries. MRI- scan shows
no brain pathologies. Thyroid
128. During regular stimulating hormone is with in
examination of a 2-year old normal limits. Serum prolactin
boy, he presents with enlarged is high. What is the most likely
left kidney, painless on diagnosis?
palpation. The right kidney was
undetectable on palpation. A. Hypothyroidism
B. Hyperprolactinemia
Excretory urography shows no
C. Poly cystic ovaries
contrast on the right.
Cytoscopy detected D. Pituitary adenoma
E. Sheehan’s syndrome
hemiatrophy of the urinary
(postpartum hypo pituitarism)
bladder trigone, the right
ureteral orifice is not detected.
What pathology is it?
130. A 45-year-old woman
A. Hypoplasia of the right came to the maternity clinic
kidney B. Dystopia of the with complaints of periodical
right kidney C. Agenes is pain sin her mammary glands
of the right kidney D. that start 1 day before
Agenes is of the right ureter menstruation and stop after
E. Ectopic right ureteral orifice the menstruation begins.
Palpation of the mammary
glands detects diffuse nodes
129. A 30-year-old woman predominantly in the upper
complains of milk discharge
outer quadrants. What is the C. Mastitis
most likely diagnosis? D. Fibro cystic mastopathy
E. Breast cyst
A. Hyperprolactinemia
B. Breast cancer

Thank You!!!

Never Give up!!!

You might also like